Этого треда уже нет.
Это копия, сохраненная 22 мая 2019 года.

Скачать тред: только с превью, с превью и прикрепленными файлами.
Второй вариант может долго скачиваться. Файлы будут только в живых или недавно утонувших тредах. Подробнее

Если вам полезен архив М.Двача, пожертвуйте на оплату сервера.
Тред тупых вопросов №100 451413 В конец треда | Веб
Тред вопросов о жизни, Вселенной и всём таком.

Спрашиваем то, за что в других местах выдают путёвку в биореактор. Здесь анонимные учёные мирового уровня критически рассмотрят любые гениальные идеи и нарисованные в Paint схемы.

Предыдущий тут: >>442222 (OP)
https://2ch.hk/spc/res/442222.html (М)

Q: Можно быстрее?
A: Можно упасть в пузырь альбукерке, наса уже почти надула его.

Q: Я начитался охуительных историй про уфологию, че делать, нам жопа?
A: Да, тебе жопа, можешь сгонять в зогач или куда оттуда пошлют.

Q: Что будет с человеком в вакууме без скафандра / если он упадет на черную дыру / попробует ступить на поверхность газового гиганта/солнца?
A: Он умрёт.

Q: Почему бы не привязать ракету к воздушному шару или стартовать с горы?
A: Космос - это не как высоко, а как быстро, большая часть энергии ракеты уходит на разгон вбок.
Подробнее тут https://what-if.xkcd.com/58/ (английский) https://chtoes.li/orbital-speed/ (перевод)
image.png584 Кб, 750x541
2 451415
>>1413 (OP)
Ебанный стыд...
Во-первых, Алькубьерре.
Во-вторых, не упасть, а создавать вокруг корабля изнутри (иначе кина не будет).
В-третьих, НАСА искривляет пространство на десятимиллионную часть, контролируя это сверхточными интерферометрами, до самого варп-привода здесь - как до Антарктиды раком.

Освятил юбилейный. Хоть и не я перекатил, как хотел, перекат получился даже лучше, чем я представлял. Спасибо тебе и вам всем, постоянным читателям раздела.
3 451425
>>1413 (OP)

>Pale Blue DotCo[...].jpg


Мда Джуно как всегда опозорилась и не смогла сделать фото Земли с места миссии
4 451429
>>1425
А ещё у них Земля круглая! Они чё по шару ходют, шароебы.
PIA12773.jpg51 Кб, 1040x1040
5 451452
Нашёл по случаю юбилея наверное самый старый из сохранившихся ТТВ:
http://arhivach.tk/thread/8170/
6 451497
>>1452
в каком году /spc заебашили?
7 451500
>>1497
Это надо у олдфагов спрашивать, я тут только с 2014-го.
sage 8 451501
Как я и опасался, коллажист совсем забыл про New Horizons. Неиллюзорной сажицы годному треду, например.
9 451504
>>1501
А я и не забыл. Пролёт мимо Ультима Туле 1 января 2019 года в 8:33 мск, к событиям 2018-го он отношения не имеет же.
sage 10 451516
>>1504

>8:33 мск


Такое себе маняоправдание.
1350191370kradenoesolnceoskazkaxru.jpg23 Кб, 250x308
11 451562
Привет спейсач. Много где слышал что от Земли до Солнца 7 световых минут. И постоянно рассказывают про то что "отключи Солнце и мы еще 7 минут будем на него смотреть и радоваться". Я чего-то задумался, а что если и правда Солнце спиздят? Что будем происходит с Земляшкой и Солнечной системой?
1)Что будет с атмосферой, морями океанами, приливами?
2)Станет ли Юпитер альфа волком и будем ли мы для него очередным спутничком или все разъебенит хуй пойми куда?
12 451563
>>1562
1. Замерзнут. В т.ч. атмосфера через лет двадцать выпадет кислородным дождем.
2. Не станет, чтобы стать спутником Юпитера надо очень удачное стечение обстоятельств - схожая траектория и пересечение орбит с существующими спутниками чтобы произошел захват, иначе будет мимопролет с искажением траектории.
13 451564
>>1563

>выпадет кислородным дождем.


охуеть
То есть Земля стала бы планетой-бродягой вместе со всеми остальными планетами нашей системы и просто ебошила в космическом пространстве без определенной траектории и прочих ненужных заморочек?
14 451565
>>1564
Да, если солнце бы пропало, так и случилось бы.
Зрелище, кстати, было бы впечатляющим: сперва внезапно наступает непроглядная ночь, потом перестает быть видным Марс, затем Юпитер и т.д. А если это произойдет когда одновременно луну видно - наступает внезапная ночь, а через пару секунд гаснет и луна.
И летите в темноте освещаемые только электричеством и начинается самая сильная зима для всех континентов. Выживут только те, кто забурятся поближе к геотермальникам и будут ходить в масках наружу с ведрами - собирать жидкий кислород.
1711256116.jpg39 Кб, 485x391
15 451566
>>1565
Вот бы сай фай такой сняли!
А Луна же отражает солнечный свет, ее вообще видно будет без Солнца? И у нас все равно будет несколько теплей чем -273, ведь ядро чего-то дает?
16 451567
>>1566
Бля перечитал>>1565 и понял что про Луну хуйню спросил не будет ее видно. Не будет видно ничего кроме звезд через какое-то время как я понял.
17 451568
>>1566
Так сняли, есть передача, но я почему-то не могу найти ее, может другой анон подскажет. Там буквально показан выход на поверхность с ведрами за кислородом.

>А Луна же отражает солнечный свет, ее вообще видно будет без Солнца?


Вот ты и ответил на свой вопрос.

>И у нас все равно будет несколько теплей чем -273, ведь ядро чего-то дает?


Будет теплее потому, что во-первых, в космосе на 4 градуса выше абсолютного нуля, во-вторых, охлаждаться излучением это очень долго, а у нас еще тектоника живая.
>>1567
Все так.
18 451569
>>1568
Спасибо за ответы, анон, теперь в голове сложилась картинка.
19 451570
>>1569
Не уходи, кстати, вдруг кто передачу вспомнит - посмотрешь с восторгом.
20 451571
>>1570
Попозже обязательно загляну и посмотрю. И сам погуглю.
21 451572
>>1571
Можешь хуевисоса глянуть на эту тему: https://youtu.be/rltpH6ck2Kc
23 451578
>>1574
Мурашки по анусу прошлись.
Но это не то, про что я говорил, вроде была передача от натгео, дискавери или ББС про протухшее солнце.
Или я уже туплю и нафантазировал себе офигенную часовую документалку из висосного видео.
24 451591
Смогу ли я голым пробежать по марсу хоть пару метров?
25 451593
>>1591
Да, и быстро потеряешь сознание.
26 451596
>>1593
Шо будет если я буду справлять НГ на Титане, взрывать там салюты и подожгу метановую реку по пьяни, какие подводные? Все полыхнет даже при местных -170?
27 451597
>>1596
Ниче не полыхнет. Что могло окислиться - окислилось.
Метан не горит без кислорода или другого окислителя.
28 451606
>>1566
Есть даже компьютерная игра. Аркс Фаталис называется. Только там поверхность покажут всего раз и то перископом, как статичную картинку, выйти туда нельзя.
1526057594844.jpg642 Кб, 1896x1008
29 451632
Что произойдет с телом человека на Венере? Допустим он стоит на фантастической венерианской базе в шлюзе и этот шлюз не резко, а медленно открывается впуская атмосферу Венеры. Человека придавит? Расплавит? От чего он умрет раньше и что станет с телом?
30 451634
может такое быть, что где-то в поясе Койпера летает бывший спутник Нептуна, который выкинул из родного дома Тритон?
31 451650
>>1634
Да, это Плутон.
32 451664
Подскажите, допустим я прилечу на луну, открою скафандр, и умру, соответственно сразу. Моё бездыханное тело будет вечно лежать на поверхности луны и потомки смогут разглядывать меня в какой-нибудь Google Moon, и мощные любительские телескопы? Ведь бактерий нет, атмосферы нет, влаги нет. Что будет разрушительно воздействовать на мой труп в скафандре?
33 451666
>>1664
У тебя внутри бактерий кишмя кишит.
Похавают немного, но в целом мумифицируешься нафиг.
В телескоп не увидят, наземные так не могут.
34 451668
>>1666

>У тебя внутри бактерий кишмя кишит.


Так они же умрут сразу, кто-то без кислорода, кто-то от температуры.
6 (5).gif8,2 Мб, 400x224
35 451669
Чё-то не могу понять, как влияет расположение суш/вод на глобусе планеты с учётом её её наклона и положения относительно местной звезды на климат планеты в целом?
Для удобства берём как всегда Как-Землю на орбите Земли в системе Как-Солнца.
Самое простое - берём и поворачиваем карту мира зеркально сверху вниз с сохранением всех параметров рек гор и прочего. Солнце по прежнему встаёт на востоке и, например, Япония по прежнему самая восточная страна, только теперь расположена в южном полушарии. В зимнее солнце стояние планета как и у нас находдится ближе всего к солнцу и повёрнута к нему южным полушарием.
36 451670
>>1669
https://what-if.xkcd.com/10/
Просто так на такое не ответить, если я правильно тебя понял.
Scheme-showing-the-earths-orbit-around-the-sun-The-distance[...].png101 Кб, 850x461
37 451671
>>1669
Земля относительно Солнца в течении года поднимается и опускается на орбите
38 451672
>>1668
Типа если ты маску шлема откроешь, то у тебя сосуды на лице и в легких лопнут, не сможешь дышать, ослепнешь, умрешь так сказать с головы, но сосуды на ногах не повредятся, пищевод уцелеет. Повреждения будут внутренними таким образом. Лицо замерзнет и превратится в льдыху, а если потом его будет освещать солнце, то растает. Еще зависит от скафандра, может нижняя часть тела будет еще долго экранирована от холода и тепла, будет гнить по земному.
39 451673
>>1632
Человек задохнется через несколько десятков секунд, труп через несколько десятков минут запечется и обуглится, далее органика скорее всего испарится, а кости и зола останутся лежать на поверхности.

Движения воздуха у поверхности практически нет, как и сильных окислителей, лютый пиздец с кислотными дождями и суперротацией атмосферы происходит в десятках километрах выше, так что ошметки должны пролежать на поверхности довольно долго.
40 451675
>>1673
А можно на Венере пирожки печь в гермошлюзе? Тесто выставляешь на противне в герму, позволяешь чуть чуть прожариться и заносишь?
image.png1,6 Мб, 1024x768
41 451676
>>1675
Можно. Главное не пережарить.
42 451736
Вопрос-викторина для людей из прошлого. Вы должны предложить особое место в Солнечной системе, где можно спрятать объект кубической формы с гранями 10x10x10 метров, который никто и никогда не должен обнаружить и затем не должен достать этот объект.

Против вас по сложности выступает вся человеческая цивилизация уровня 2118 года, так что добраться в некие далекие отдаленные места Солнечной системы они могут. Куда бы вы спрятали КУБ, так чтобы его не нашли, а если даже найдут - не смогли достать, но так чтобы он не был полностью утерян. Это исключает ядра звезд и планет гигантов разумеется.
В.В. Путин - кто первым полетит на Марс.mp46,1 Мб, mp4,
1280x720, 0:48
43 451737
https://tass.ru/kosmos/5964120

>На биологическом спутнике на рекордную высоту в 800 км будут запущены 75 мышей. Основной акцент в исследованиях сделают на влиянии космической радиации. "Биологические объекты будут впервые на такой высоте


>Заявок у нас где-то к полусотне. В исследованиях хотят участвовать американцы, немцы, есть предложения от китайских и малайских коллег



Зачем нужна такая международная программа, если у НАСА есть данные сотен кхм.. человеко-дней астронавтов в условиях жёсткого космического излучения?
44 451741
>>1737
Потому что статистика лишней не бывает. Потому что методы диагностики немного изменились.
Потому что сотни человеко дней это нихуя.
1.jpg82 Кб, 490x694
45 451745
Это тред тупых вопросов?

Почему человек в скафандре, а заяц без скафандра?
46 451746
>>1745
Очевидно же что заяц это робот, где ты в реале видел улыбающихся зайцев?
2.jpg74 Кб, 395x356
47 451747
>>1669
Во-первых, в получении солнечной энергии между Северным и Южным полушариями имеется заметная асимметрия. См. пикрилейтед.
48 451751
>>1745
На пикче - полет Союз-11. Заяц, ёлка, письмо и внешний вид "ракеты-космического корабля" - предсмертные галлюцинации космонавтов, вызванные кислородным голоданием из-за разгерметизации капсулы.
ты меня расстроил.jpg81 Кб, 535x528
49 451752
50 451756
>>1736
В точку лагранжа нептуняши.
51 451757
>>1736

>Куда бы вы спрятали КУБ, так чтобы его не нашли, а если даже найдут - не смогли достать, но так чтобы он не был полностью утерян.



В дамскую сумочку своей тян, где-то между кремом для рук и маникюрными ножницами.
Стикер512x512
52 451758
>>1757
О, я тоже спрятал бы в несуществующую сумочку вымышленного человека.
53 451759
>>1747
Абажжи, ёбанырот.
Но ведь южное полушарие получается теплее. Джунгли Амазонки, южная половина Африки, Австралия с Океанией и всё такое.
Это не к нам на севера прилетают греться жители южного полушария.

> Во-первых,


А во-вторых?
HemisferioSur.png43 Кб, 1000x442
54 451760
>>1759
Ты на широту-то Австралии с Океанией посмотри. И сравни с широтой своего города. Вот карта: белое - северное полушарие, жёлтое - южное, между ними экватор.
Кроме того, так получилось, что в Южном полушарии куда больше воды. Это сильно смягчает климат.
Короче говоря, климат бы значительно изменился.

> А во-вторых?


Во-вторых, например, орбиту Луны ты собрался на месте оставить или тоже перевернуть? Она оказывает на Землю и её обитателей много разных воздействий - от приливов (которые, кстати, бывают не только на море: когда над тобой проходит Луна, ты вместе со своим домом к ней поднимаешься примерно на 50 метров - приливы в земной мантии!) до женских месячных.
55 451762
>>1741
А как они защитили людей от радиации за пределами магнитного поля Земли? Почему эти чудо материалы теперь нигде не используют?
56 451763
>>1762
Они полторы недели были за магнитным полем, это не на марс летать.
57 451765
>>1760
Пиздос. Получается, между южной пипкой Южной Америки и мысом Антарктики так много воды? Сколько там воды в линейном километраже? Если можно, в сравнении с аналогичным расстояниями где-нибудь в Европе, чтобы понять было проще.

Не, орбита Луны, её вращение и всё прочее остаются неизменными.

> от приливов до женских месячных


Ну это-то не секрет
58 451767
>>1758
Эх, если бы вымышленного. Женат 10 лет.
Безымянный.jpg58 Кб, 800x600
59 451768
Можно ли взлететь в космос на гребне огромного цунами, или не получится нихуя?
60 451771
Что за традиция брызгать на колесо автобуса? А как женщины в этом участвуют? У крыла приседают?
61 451772
>>1771
Как и полагается суеверной хуйне - в ней нет никакой логики, не ищи ее в таких традициях.
62 451777
>>1768
Нет, т.к. ты не учитываешь, что не сможет получиться волный такой высоты и силы, что тебя на другой стороне планеты вытолкнет на орбиту или хотя бы за атмосферу. Об столкновении с астероидом/планетоидом не мечтай - при таком ударе вода разлетиться прочь прямо возле удара (да и планетка треснет). То, что ты хочешь, при определённых условиях таки возможно, но это должна быть более густая и вязкая жидкость, чем вода.
236547u54i8.jpg761 Кб, 1534x1652
63 451780
>>1413 (OP)
Всели звёзды существуют только в скоплениях - галактиках, или звёзды имеются и в пространстве между галактиками? Существуют ли звёздные системы в межгалактическом пространстве?
64 451781
>>1768
Физика волн сложнее, там волна будет взаимодействовать с островами, более глубокими и мелкими местами и до другой стороны планеты дойдут отдельные мелкие волны с разных направлений.
65 451782
>>1780
Конечно, есть.
66 451784
>>1736
Да просто покрасить в черный цвет и на случайную орбиту выкинуть подальше от Солнца, неплохо бы еще и с большим наклонением, даже если обшаривать всего лишь каждый миллиард кубических километров, найдут году этак к 211111111118.
67 451785
>>1765
Если с картой России сравнивать: мыс Горн на месте Москвы (по широте практически идеально совпадает), кончик Антарктического полуострова — на месте Архангельска.
68 451787
>>1784
тут в задаче два условия ты пропустил - нужно закинуть не в случайную орбиту, а в определенную, где ты при желании мог бы его обнаружить, зная где он находится, но также чтобы это вызвало определенные сложности у человечества в том, чтобы достать его, если они его обнаружат. То есть у задачки первая сложность обнаружение и вторая сложность возвращение. Вы с этим аноном >>1756 вторую сложность забываете.
world[1].gif405 Кб, 1957x987
69 451790
>>1785

> Расстояние Архангельск Москва по трассе составляет 1227 км, а по прямой - 991 км.


Хм.
Взял карту, прикинул размеры -какая же Земля маленькая, пиздец просто. Космическая деревня просто.
70 451791
Если мы на Марс привезем контейнер с грузом 10кг, то на Марсе он будет весить меньше приблизительно 4кг. Как колонисты на Марсе будут ориентироваться между этими двумя цифрами? Они будут говорить что объект весит 4 марсианских килограмма, или будут говорить что весит 10кг земных, просто они будут понимать что они легче?
71 451792
>>1791
Масса-то не меняется от гравитации.
Масса будет 10кг.
У нас на земляшке тоже не везде идеальные, колеблются от 9,79317 м/с2 в Каире до 9,81927 м/с2 в Осло.
Для этого весы и калибруют.
На Марсе весы будут откалиброваны так, что объект массой в 10кг будет показан на весах как 10кг.
Просто все будут понимать, что один человек без проблем может посылку массой в 50кг поднять без риска.
72 451800
https://www.youtube.com/watch?v=hLHo9ZM3Bis
Так это что получается, у шаттла был не абляционный теплощит?
73 451831
Сможет ли земная ворона в скафандре летать на Марсе?

мимо-НЕ-ворона
NIR1.jpg289 Кб, 642x1083
74 451833
>>1800
Ну здрасьте, а ты всё это время думал, что абляционный? Тогда бы пришлось после каждого полёта вообще всю теплозащиту менять, а не только повреждённые плитки, Шаттл по цене был бы не золотой, а родиевый нахуй.
75 451834
>>1760

> когда над тобой проходит Луна, ты вместе со своим домом к ней поднимаешься примерно на 50 метров - приливы в земной мантии!)


Ето правда?
76 451838
1. Удельный импульс всегда постоянный и не меняется в РДТТ в зависимости от увеличения площади горящего топлива? Увеличивается сила тяги, но пропорционально увеличивается и масса выхлопа, так?
Пример: если мы уберем сопло и дадим цельнолитому топливу медленно и печально гореть с конца - или если поставим хорошее сопло Лаваля, а топливо в движке вообще сможет сгорать килограммами в секунду (имплаинг, что корпус движка из мифрила и все выдержит) - Isp будет один и тот же?

2. На удельный импульс вообще что влияет? Только теплота горения топлива? Сопло, как я понял, может только увеличить тягу.

3. Есть ли конечная простая формула (аналитическое решение, без интеграции) типа уравнения Циолковского, но учитывающая также сопротивление воздуха, его изменение от скорости, изменение силы тяги в зависимости от изменения площади горения (о, как) - чтобы выдать расчетную предельную скорость ракеты в воздухе? Или все же надо интегрировать в цикле по dt?
77 451839
>>1834
Почти: насчет размера он припизднул в 100 раз, не 50 метров, а 50 сантиметров, но само явление действительно существует.

https://en.wikipedia.org/wiki/Earth_tide
78 451845
>>1838
1. меняется в зависимости от формы сопла, особенно расширения газов и скорость их выхлопа

>Пример: если мы уберем сопло и дадим цельнолитому топливу медленно и печально гореть с конца


Тогда вообще импульса не будет так как скорость истечения будет никакая

2. у химических параметры топлива, сопло и немного схема работы
3. хуй знает откуда ты вообще площадь горения выдумал
PzDnY.gif4 Кб, 553x225
79 451851
>>1845

>Тогда вообще импульса не будет так как скорость истечения будет никакая


А вот не уверен. Isp = Thrust / (Mass flow rate x g)
Там выброс массы будет никакой, но и тяга пропорционально никакая. А удельный импульс будет тот же.

3. Пикрил. А еще ведь когда и если тяга на какой-то скорости сравняется с сопротивлением, и больше не вырастет, это и будет терминальная скорость. И может даже заметно меньше, чем по формуле Ц.
81 451887
>>1886
Ты бана хочешь что ли?
Зачем ты эту жирноту вбросил?
82 451889
>>1851

>Isp = Thrust / (Mass flow rate x g)


Хуета какая то. Импульс это скорость истечения газов разделить на g.

>А удельный импульс будет тот же.


Нихуя там не будет потому что скорость истечения газов свободного горения никакая

>Пикрил.


Ты хоть смотри что ты кидаешь, это для гибридных ракет, т.е. вот эта хуйня и есть сопло по сути
83 451890
>>1833
Я думал он просто жирный, поэтому его не каждый раз меняли. А как тогда он работал? Тупо теплоизоляция?
84 451913
Космоаноны, обоснуйте такую тему.
В околокосмических фильмах часто показывают, как люди, находящиеся по факту либо в открытом космосе (вакууме), либо на небесных телах с сильно разряженной атмосферой (Луна, Марс и т.п.), при наступлении "ночи", в считанные минуты начинают замерзать. В контексте таких сцен обычно говорится, что ночью на Марсе -150 градусов, поэтому и замерзают.
Однако, какого хера они почти что в вакууме мерзнут? Там же теплообмена нет, только через туфли об землю разве что. А излучением человеческое тело теряет очень мало тепла, всяко меньше, чем оно способно выделять в процессе обычной жизнедеятельности. Какого хера конструкторы скафандров на такой случай не додумались напихать в скафандры фольги, чтобы и это самое излучение удерживать, тем самым практически полностью исключив проблему возможного замерзания космонавта.
85 451915
>>1913
Фильмы про космос чаще всего снимаются даунами, которые никогда не были в космосе,ничего про космос не знают и не знают физику. Представь если бы фильмы о море, о шторме, о путешествиях на морских кораблях снимались бы монгольскими коневодами, которые никогда в море не были и никогда в жизни не были на корабле. Такие фильмы кишели бы неточностями и непониманием как же на самом деле работает море, какой быт на кораблях и как корабли управляются, так как монгольские коневоды не имеют понятия как эти вещи работают. Так вот фильмы о космосе снимаются людьми, которые также никогда не были в космосе, которые не имеют понятия как космос и космические корабли работают.

>Какого хера конструкторы скафандров на такой случай не додумались напихать в скафандры фольги


В реальных скафандрах есть теплоизоляция лучше, чем фольга. В воображаемых скафандрах в космосе я не знаю что они туда пихают.
86 451950
Как на МКС стирают вещи?
87 451952
>>1950
Не стирают, носят вещи свежие, потом в пакет и сжигают в атмосфере на прогрессах.
88 451959
>>1889
Алло, это его основная формула. Твоя только вторичная. Если угодно скорость истекающих газов 1 метр в секунду = 1 ньютон тяги / расход 1 килограмма топлива в секунду (g сокращаем)
Или сколько секунд 1 кг сгоревшего топлива будет выдавать тягу в 1 ньютон. Одно и то же.

Это не сопло, а сечение топлива, так что скорее "камера сгорания". Сопло уже на выходе.
89 451962
>>1952
как же там воняет, наверное
90 451983
>>1962
Там все стерильно.
91 451990
>>1983
Люди тоже стерильные?
92 451998
>>1983
Настолько стерильно, что бактерии даже по внешней поверхности МКС уже ползают.
93 452013
>>1998
Это снаружи налипли
94 452019
Можно ли на МКС курить?
95 452025
>>2019
Только в форточку.
96 452066
>>1983
Как там может быть стерильно, если там люди живут, лол?

>Бактерии поселились на Международной космической станции с 1998 года, когда ее еще только монтировали. И прекрасно приспособились к тяжелым условиям в космосе, в том числе к лютому холоду, отсутствию воды и даже к радиации. В общем, стали мутантами. За прошедшие годы были выявлены 76 разновидностей микроорганизмов, в том числе весьма вредоносных. Причем попытки извести их к успеху якобы не приводят - не помогают ни антибактериальные жидкости, ни тщательные проверки экипажей и кораблей перед отправкой в космос

изображение.png153 Кб, 320x213
Сколько будет светить Бетельгейзе 97 452073
Вот взорвется новая звезда. Допустим, Бетельгейзе. Будет у нас две звезды, ночи не будет. А сколько это будет длиться? И что, будем 5/50/500 лет жить с двумя солнцами?
Comparativesupernovatypelightcurves.png137 Кб, 1187x768
98 452080
>>2073
От пары месяцев до пары лет.
99 452086
>>2073

>Будет у нас две звезды, ночи не будет.


Такого не будет, твердо и четко
21761900.jpg237 Кб, 1741x571
100 452094
Поясните зачем такое делают. Я еще понимаю Оумуамуа, сломал язык и все таки запомнил, но это какой-то садизм и дебилизм. Еще ведь и название со звуками, которых наверное в других языках нет и которые не произносятся. Зачем как-то возносить папуасов из намибии, которые ничего не достигли?
101 452095
>>2094
Видимо, кому-то показалось это красивым: https://upload.wikimedia.org/wikipedia/en/3/3a/Gǃkunǁ'homdima.ogg

Алсо это название есть только на англоязычной версии вики. В других - только каталожное обозначение. Причем в каталогах я не нашёл упоминания этого названия "Кункгхудима"
image.png14 Кб, 454x87
102 452096
>>2095

>Кункгхудима

103 452097
>>2096
Это я попытался человеческим языком написать то дебильное сочетание несочитаемых символов
104 452098
>>2094
Насмотрятся черных пантер, а потом поклоняются папуасам. Период такой сейчас, кали-юга на марше.
maxresdefault.jpg110 Кб, 1280x720
105 452105
>>2094
Если всяким regressive left покажется, что ты недостаточно уважаешь папуасов, то в твоей карьере могут возникнуть проблемы.
106 452160
Правда, что у КА с ритегами качество фото лучше, чем у оснащённых солнечными панелями?
107 452162
>>2160
Да
108 452163
>>1745
Проходит эксперимент по влиянию космического вакуума и излучения на живые организмы. В то время эта область была слабо исследована, приходилось ставить опыты на животных. Собственно, космонавт проводит эксперимент и записывает наблюдения, а заяц является "подопытным кроликом".
109 452165
>>2160
Взаимосвязи нет.
110 452167
>>2165
Это только ты так думаешь
111 452168
>>2167
Это факт. Оптический инструмент может быть совершенно любой и "качество фото" зависит от него, а не от источника питания.
112 452169
>>2168
Создание космической техники это не 2+2, это куча взаимосвязей и сложностей. Отсутствие прямой связи не означает полное отсутствие связи.
113 452171
>>2169
Доказывай, где связь между источником энергии и качеством фото.
Но сперва дай определение фразе "качество фото".
114 452172
>>2171
Ты просто доебываешься уже
but why.mp443 Кб, mp4,
480x270, 0:02
115 452173
>>2172
В /б/ забанили что ли? Шитпостить негде?
116 452174
>>2173
Заебал, просто сравни реальные аппараты. Кроме мро нихуя нет нормальных камер, и то мро узкоспециализированный аппарат.
117 452176
>>2174
Хаббл не космический аппарат?
Джеймс Уэбб не космический аппарат? инб4 нет, он еще назимле))
Чандра не космический аппарат?
И тут же вопрос про "фото" потому, что фото в привычном понимании они не делают, пикчи которые мы видим на сайте насы как и практически все из космоса сшиваются из равок которые чаще чем в большинстве случаев снимают не только в оптическом диапазоне.
И почему надо было пояснить про "качество фото", как этот критерий-то расценивается? Что качественнее - экстремально глубокое поле Хаббла или близкое фото атмосферы Плутона?
Это сравнивать нельзя, обе вещи (и все остальное) снимается специализированными инструментами разработанными под задачи миссии.
Так что все "фото" максимально качественны вне зависимости от источника энергии, они максимально качественны в зависимости от целей миссии и от бюджета.
118 452187
>>2176
Бля очевидно что аппараты на орбите земли и тем более телескопы учитывать не стоит
Речь про дальние аппараты, которые несут много экспериментов и камера лишь часть из них и не является основным. И вот тут аппараты с панелями полностью сосут у ритэговых.
119 452189
>>2187
Сосут не из-за оптики а из-за неэффективности солярок.
Для полета к Солнцу надо не меньше дельты, чем для дальнего полета, но для ближних солнечных исследований и меркурия вполне хватит солярок, даже с запасом.
120 452196
>>2189
Ну вот мы и пришли к выводу что по тем или иным причинам аппараты с солярка имеют не самые лучшие камеры. На Джуно вообще камера уровня мобилы.
121 452197
>>2196
Нет, по тем причинам, по которым тебе было удобно классифицировать, напрочь проигнорировав месседж тут >>2176
>>2187

>тем более телескопы учитывать не стоит


Вот это вот яскозал и дальше удобно ритеги выставить, угу.
С хуев ли крашеных, скажи-ка, орбитальные обсерватории не стоит учитывать?
И с хуев ли камеры имеют какое-то значение, если они на каждый аппарат изготавливаются абсолютно индивидуально под параметры миссии?
Джуно несмотря на "уровень мобилы" передал охуительнейшие сведения о Юпитере.
image.png670 Кб, 945x473
122 452312
Когда ядро звезды коллапсирует в черную дыру, куда деваются эти милиарды трилионов кельвинов? У сингулярности что есть температура? А заряд электрический?
123 452355
>>2312
ПОВСЮДУ.
Сверхнова это одно из ярчайших событий во вселенной.
И кучу материи раскидывает вокруг.
124 452358
>>2312
Энергия переходит в массу.
125 452359
>>2312
В энергию вращения и гравитационные волны.
Теоретически есть типы чёрных дыр с зарядом.
126 452361
>>2312
Как обнаружили Бекенштейн, Хокинг и др., для черных дыр также справедливы законы термодинамики. Поэтому, согласно первому закону термодинамики, при коллапсе ничего никуда не девается - все, что не может быть излучено в виде электромагнитного/нейтринного/гравитационного излучения, переходит в массу, электрический заряд и момент вращения черной дыры.

Говорить о черной дыре как о сингулярности с точки зрения внешнего наблюдателя нельзя по крайней мере в рамках общей теории относительности так как сингулярность находится за горизонтом событий, а значит, внешний наблюдатель принципиально не может ее увидеть - свет из-под горизонта не может достигнуть наблюдателя.

Площадь горизонта событий в термодинамике черных дыр пропорциональна их энтропии. Энтропия горизонта много больше энтропии любой материи, из которой образуется дыра с данной площадью горизонта событий, поэтому второй закон термодинамики также справедлив для черных дыр.
127 452364
Сложный, наверное, а не тупой вопрос будет.

Посмотрел лекции всяких астрономов и иже с ними и узнал, что Земля спокойно могла засрать жизнью всю в солнечную систему без всякого нашего участия.

На стеклах иллюминаторов мкс находили бактерии, которые не могли быть доставлены миссиями, т.е. они прилетели сами, при любом проходе потока метеоров множество их проходит через атмосферу и не падает, а отскакивает и улетает дальше, какие-то бактерии выжили на американском зонде, который потом забрали члены миссии одного из аполлонов, фактически на любом теле в солнечной системе можно найти годные условия для жизни (на Луне на глубине метров 5, на Венере в верхних слоях атмосферы, на Европе в подледном океане).

И как то от всего этого у меня упал хуй на поиск инопланетной жизни, ведь если даже что и найдут - вероятность того что это с Земли улетело в свое время чуть меньше чем 100%.

Кто-нибудь знает про это профессионально и побольше? Есть шанс отличить? Получается какую жизнь не найди - это не "бог умер", а "аа, епта, это во времена динозвров метеорит с Земли выбил" - неиллюзорный такой вариант.
128 452366
>>2364
Для этого есть ОТДЕЛ ПЛАНЕТАРНОЙ ЗАЩИТЫ, который вопреки названию защищает не землю. Все зонды стерилизуют и расчитывают на риск и теоретические возможные микроорганизмы которые могут быть нанесены зондами.
https://www.nasa.gov/planetarydefense
Яркие радужные новости по открытию тихоходок в космосе никто всерьез не будет публиковать, большинство будет всерьез полагать что эта тварюга с Земли налетела.
129 452367
>>2364

>Есть шанс отличить


Сравнить генетический материал с таковым у земных организмов, очевидно же.
130 452380
Если на Луне оставить гору пластиковых бутылок, то сколько времени они будут разлагаться?
131 452382
Когда гравитацию смогут?
132 452383
>>2380
Зависит от пластика, время разложения ПЭТ тары под ультрафиолетом - тоже годы, как и на земле, но чуть быстрее из-за открытого УФ и большего нагрева.

>>2382
Уже. Или ты летаешь по орбите и уже соскучился?
133 452384
>>2380
Будут валяться, пока их ультрафиолет молекулы не разъебет на куски. Думаю, что пару лет, а потом рассыпятся.

мимо работаю с пластиками
134 452385
>>2383
Я про то, что когда выяснят ее природу и все такое.
135 452387
>>2384

>мимо работаю с пластиками


О, привет.
Скажи, пожалуйста, как меняется состав этанола хранящегося в ПЭТ таре при 40 градусах целься на протяжении полугода?
ПЭТ-таре пофиг (верно?), а каково будет употребившему продукт из неё?

>>2385
Ванги, к сожалению, в отпуске из-за новогодних праздников, будущее здесь не предскажут.
Пока что ньютоновская механика вполне работает в быту.
ОТО нормально объясняет механику искажения пространства-времени для массивных объектов ака гравитацию. Могу посоветовать глянуть пару видео на эту тему.
136 452392
>>2387

>О, привет


Я не химик, но конструкционно пэту похий на этанол. Может в нем вечно храниться. Но как самогонщик, рекомендовал бы хранить в стекляшках, если у тебя какие настойки. Этанол не разъедает пластик, но какие-то невкусные ноты все равно добавляет.
137 452395
Если на мкс высунуть жопу в герметичную дыру, вакуум высосет говно из жопы?

Говно будет арбить или уподет на землю?
138 452396
>>2392
Разъедает. ДОООЛГО и МЕДЛЕННО. Но ПЭТ при положительных температурах теряет целостность в целом и даже больше при воздействии этанола.
Мне сейчас лень бумаги искать, но вкратце этанол рассматривался как катализатор для утилизации ПЭТ, при 400 градусах и 2 сутках воздействия ПЭТ очень хорошо разлагался.
Факт есть в том, что ПЭТ даже при небольших температурах с этанолом реагирует и вопрос в составе этанола после этой реакции.
139 452397
>>2395
Сфинктер нормально выдержит, если у тебя не понос. Кроме шуток.
Твердые каловые массы удержатся сфинктером, жидкие или полужидкие выдавятся давлением тела и атмосферы МКС.
Фекалии останутся примерно на той же орбите и с немалой вероятностью очернят солнечные батареи.
140 452399
>>2396
Не уверен, но тот анон вроде как практический смысл спрашивал. В темном помещении и при комнатной температуре бухло можно хранить хоть бесконечно.
141 452400
>>2399
Этанол таки влияет на ПЭТ. Взаимно.
Можешь нагуглить или подождать пока мне станет не лень сделать это для тебя.
Нержавейке похер. Стеклу - похер. ПЭТ - "похер".
Тара будет целой долгие годы, а вот содержимое - не будет.
142 452418
>>2397
>>2395
А разве давление воздуха внутри станции не будет давить? Не превратится ли человек с голой жопой, засунутой в космическую форточку в воздухоотсос? Так-то, человек - сквозной отверстие. Воздух войдёт в рот, пройдут весь ЖКТ и выйдет из анального ануса сфинктера жопы. Сомневаюсь, что клапаны внути ЖКТ человека сильнее космического вакуума.

У меня, кстати, тоже есть тупой вопрос. Почему в космосе за орбитой Плутона так много картофелиоидов?
То комета Чурумова-Герасименко, то вон так штука, которую сфоткал Горизонты на Новый Год - закономерность что ли?
Как вообще образуются картофелиоиды и почему они не образовываются сфероидами? Чому тогда спутники Плутона-Харона что ни тело, то ни сфера, ни картошка, а так - ни рыба, ни мясо - рисинки какие-то.
Вроде, картинки достаточно разношёрстные, чтобы привлечь внимание
images.jpg4 Кб, 312x161
143 452420
>>2418

>15464425886291.jpg


надо было жопку протона постить
144 452421
>>2418
У них нехватает массы принять форму близкую к сфере.
145 452423
>>2421
Но почему тогда форма картошки, а не снежинки, например?
146 452424
>>2423
На картошку как раз хватает. Они же из частиц пыли и льда, для охуенной снежинки слишком тяжёлые и сыпучие, для шарика лёгкие. А так как они соединение нескольких глыб, получается неровная, бугристая картоха.
147 452428
>>2424
Но ПОЧЕМУ? Почему это всегда картошка из двух полужопий, одно из которых больше? Почему не три, не четыре, не девять?
244345462.jpg56 Кб, 707x433
148 452463
Почему луна и со второго китайского лунохода опять коричневая, как и в прошлый раз, а у американцев всегда была серой?
149 452467
>>2463
фильтр в инсте не тот выбрал.
Вопрос знатокам. Слышал где-то что существует Звезды (ну или трупы звезды) поверхность которого составляет 30 градусов по Цельсию.
И я где-то прочитал что Солнце после того как откинется, превратится в планетарную туманность
150 452469
>>2463
Китайские матрицы из говна.
151 452471
>>2463
может американцы садились близко к экватору - Солнце высоко в небе, высокий контраст, все засвечено к хуям, видно лишь белый и серый цвета грунта.
Китайцы сели у полюса в высоких широтах, где Солнце у горизонта и видны коричневые тона грунта, когда Солнце светит, но не засвечивает.
152 452475
>>2467
В чем вопрос-то?
153 452482
>>2463
Бля, не могли яркостной и цветовой эталон нанести на корпус лунохода. И что теперь думать?
154 452484
>>2463

>Какого цвета Луна?


https://habr.com/post/207824/
ab65d264121e4c82504219099d4e641f.jpg832 Кб, 900x909
155 452489
>>2484
Так-то луна действительно желтоватая, даже невооруженным глазом. Через атмосферу, релеевское рассеяние. Значит, без атмосферы еще желтее в среднем, все верно.
156 452495
>>2463

>https://ru.wikipedia.org/wiki/Луна


>цветовые различия на Луне крайне малы. Её поверхность имеет коричневато-серую или черновато-бурую окраску.


То есть поверхность однотонная с заметным смещением спектра в красную область. Не серая, но и не пипец какая цветная.
157 452501
>>2418

>А разве давление воздуха внутри станции не будет давить? Не превратится ли человек с голой жопой, засунутой в космическую форточку в воздухоотсос? Так-то, человек - сквозной отверстие. Воздух войдёт в рот, пройдут весь ЖКТ и выйдет из анального ануса сфинктера жопы. Сомневаюсь, что клапаны внути ЖКТ человека сильнее космического вакуума.


Разница давления всего лишь в одну атмосферу.
Мясные мешки выдерживают десяток атмосфер без ущерба для здоровья, их не выдавливает как тюбик вопреки кажущемуся.
158 452502
>>2475
Слышал ли анон про это, или нет, наверное.
159 452503
>>2489
Влияние атмосферы настолько ничтожно, что им можно пренебречь, оно никак не влияет на восприятие цвета.
160 452504
>>2489
Шизик с желтой луной из телескопотреда, ты?
161 452505
Если уронить Фобос на Марс можно будет его раскрутить как сраную земляшку и оживить тектонику ну и ядро заодно разогреть?
162 452506
>>2505
Нет, это слишком маленькая картошечка.
163 452507
>>2506
Бля. Рядом с Марсом крупных картофелин вроде нет. Цереру невозможно будет швырнуть в Марс скорее всего, только если пиздануть по ней другим бужыником и сбить с орбиты.
А если на протяжении сотни-другой лет бомбардировать поверхность камушками от ~2км? Как это в перспективе на ~1 миллион лет скажется на климате планеты?
Не фабрики же там строить ей богу.
164 452508
>>2505
>>2507
Фобос и так на него скоро ёбнется, там почти предел Роша достигнут.
Алсо, оживлять его тектонику нафиг не нужно. Постили ведь уже хитрый план с магнитом от томографа в L1, которого хватит для перекрытия магнитной тенью целой планеты.
165 452509
>>2418

>Сомневаюсь, что клапаны внути ЖКТ человека сильнее космического вакуума.


Почему сомневаешься?
166 452514
>>2506

>маленькая картошечка.


Что бы всем было очевиднее, сравнение размеров мелкого мусора на фоне Цереры, на втором пике можно сравнить с Марсом.
marsceresassteroids.jpg78 Кб, 1536x1166
167 452515
Вот как то так
168 452519
Если магниты сложить вместе, суммарный магнит получится сильнее, как сумма магнитов или слабее чем сумма?
169 452525
Аноны, вот чисто теоретически, может ли существовать такая планета которая будет вращаться вокруг своей оси со скоростью света? И если да, то какие из этого выходят подводные?
170 452536
Знатоки а что будет если ракету медленно поднять на высоту ну скажем 2000 км ? Она ебнется на землю?
Ее нельзя медленно поднять ибо законы физики/ небесная твердь?
Запускают на 100км и возвращают без проблем. А есть предел высоты?
171 452539
>>2536
Орбита это вбок, а не вверх. Без первой космической любое тело ебнется га планету.
172 452552
>>2525
Ноуп. Её разорвёт. Осилить вращение со скоростями близкими к световой могут только черные дыры и может ещё нейтронные звёзды заметный процент осилят.
173 452553
>>2536
Если постоянно ползти вверх то уползти из гравитационного колодца можно хоть со скоростью черепахи. Но как только ты отключишь движки начнёшь падать. Может на планете, может на Солнце.
174 452572
>>2553
То есть если медленно взлететь на геостационарную орбиту и выключить двигатель то ебнишся на землю? Как то слабо верится(
175 452580
>>2553

>Может на планете, может на Солнце.


Nyet. Если ты просто съебал от земляшки, то вокруг Солнца ты двигаешься почти со скоростью Земли. Что бы упасть на Солнце придется гасить почти 30 км/c.
176 452583
>>2572
Попробую объяснить на пальцах, что такое орбита.
Вот допустим ты подпрыгнул на метр, после чего упал. Это был суборбитальный полет. Ты можешь направить двигатель вниз и медленно поднятся до высоты тридцатого этажа. А потом выключить двигатель и уебать в асфальт с высоты в сотню метров. Займет падение примерно 4 секунды. Это то, что предлагаешь ты.
А орбита, это когда ты на высоте тридцатого этажа летишь над землей пиздец так быстро, что улетаешь за горизонт раньше, чем касаешься земли. И при этом из-за кривизны земли не приближаешься к поверхности а продолжаешь улетать за горизонт пока не сделаешь кругосветное путешествие.
Ну за исключением того, что на высоте 30 этажа такому помешает атмосфера, да и в гору какую-нибудь врежешься.
177 452585
>>2519

>Если магниты сложить вместе, суммарный магнит получится сильнее, как сумма магнитов или слабее чем сумма?


Очень всрато сформулированный вопрос.
Магниты (постоянные) "равномерно заполнены" полем, индукция которого зависит от материала магнита. Если слепить несколько неодимовых по 1Тл, то 3Тл не получится - будет один большой магнит с полем 1Тл. А вот сила отрыва от железной хуйни будет разной, потому что больше силовых линий пройдёт через железную хуйню.

>>2525

> Аноны, вот чисто теоретически, может ли существовать такая планета которая будет вращаться вокруг своей оси со скоростью света? И если да, то какие из этого выходят подводные?


Гугли парадокс Эренфеста.
178 452593
>>2572
Включу немного буквоеда, но думаю тебе это добавит понимания

>То есть если медленно взлететь на геостационарную орбиту и выключить двигатель то ебнишся на землю?


Если ты взлетишь на высоту геостационарной орбиты, не имея нужного вектора и параметра орбитальной скорости, то да, ты полетишь вниз.
179 452603
Аноны, а вот летит себе блекбёрд на высоте почти 30 км со скоростью тыщу метров в секунду. И каждую секунду, если он сохраняет прямую траекторию в абсолютной системе координат, он прибавляет 5.5 см высоты ввиду кривизны Земли. Но это если бы каждую секунду высота корректировалась. Однако если пилот решит поспать скажем 10 минут, то по моим расчетам прямолинейное движение добавит уже 13 км на 600 пройденных по прямой. То есть очевидно, что нужно постоянно корректировать. Вопрос - это автоматика корректирует, или пилот постоянно штурвалом правит, ориентируясь по альтиметру, если эшелон оговорен?
180 452604
>>2603
Автоматика.
А высоту он не наберет, у него движки начнут захлебываться без кислорода, упадет скорость, подъемная сила и высота.
181 452605
>>2593
Но ведь геостационарная орбита синхронизирована с землей.
По сути если смотреть с земли то там на этих сколько его там, 30 000 км высоты никто никуда не летит.
Хочешь сказать если туда долететь без горизонтальной скорости то спутники там будут летать мимо тебя со скоростью вращения земли? И как только ты выключишь двигатель ебнишся на землю в то место откуда взлетел?
NewtonsCannon.jpg220 Кб, 1920x1080
182 452612
>>2605

>если туда долететь без горизонтальной скорости то спутники там будут летать мимо тебя со скоростью вращения земли? И как только ты выключишь двигатель ебнишся на землю в то место откуда взлетел?


Именно так.

https://riko.space/extra/skorosti-v-kosmose/
183 452613
>>2605
Орбитальная скорость не равна скорости относительно поверхности. Потому что скорость орбиты считается от центра Земли. Стоя на экваторе у тебя будет около 460 м/с вроде скорость относительно центра.
Аналогично, скорость относительно центра на высоте геостационара не равна нулю.
184 452614
>>2605

>Но ведь геостационарная орбита синхронизирована с землей.


С земли ты можешь забрать только бонус линейной скорости в 400 с копейками м/с вбок. Чтоб не улететь в далекие ебеня, а подниматься четко над одной в сферических координатах точкой, тебе ее нужно погасить. Хотя она будет погашена атмосферой при медленном подъеме. Синхронизация орбиты означает, что совпадают угловые скорости. Само собой линейные скорости при радиусе кривизны 6400 км и радиусе 42000 км будут сильно разниться при равенстве скоростей угловых.
185 452622
>>2604
Да само собой я в курсе, что от характеристик набегающего потока зависит номинальная мощность, и далеко высоко он как ни крути не улетит. И очевидно, что для оптимизации полета нужно удерживать эшелон. Мне просто интересно а как часто происходит корректировка? Я так вот думаю если он бы безотрывно корректировал высоту, то есть двигался бы по сектору окружности в земной радиус, то скорость в 1 км/с давала бы уменьшение силы притяжения на где-то 1/64, что всяко солидней профита от высоты, который где-то 1/256. И тут еще возникает вопрос, существует ли разница в подъемной силе на высоких скоростях 2М+ в зависимости от движения с запада к востоку, или наоборот. Понятно, что двигаясь в среде, самолет как бы плывет в атмосфере, летя относительно нее. Но он также и относительно земной поверхности движется. И в зависимости от направления движения скорость движения по кривой окружности с центром в центре масс планеты будет отличаться почти на километр. А это уже разница между (1/14)2 до (1/5)2
186 452627
>>2614
Допустим, что атмосфера вообще не имеет течений. Значит, вращается с той же угловой скоростью, что и Земля. Означает ли это, что тот же метеозонд, поднимаясь вверх, "сам собой" увеличивает свою линейную скорость относительно некоей неподвижной точки наблюдения на орбите Земли вокруг Солнца, и при этом его линейная скорость относительно перпендикуляра к поверхности Земли все тот же 0?
И при таких условиях можно выйти на геосинхронную орбиту, не разгоняясь вбок до первой космической, а просто отключив двигатели на нужной высоте?
187 452633
>>2515
Пасиба
188 452636
>>2613

>скорость относительно центра Земли на высоте геостационара не равна нулю.


??
5t0.jpg34 Кб, 800x504
189 452637
Да, что-то странное. Или я туплю.
Все говорят: "первая космическая". А ведь она должна как бы сама собой набраться при вертикальном подъеме на нужную высоту с любой скоростью. Угловая скорость сохранилась и никуда не теряется.
190 452640
>>2636
Ну бля орбитальная скорость не равна скорости относительно поверхности
>>2637
Почему должна?
191 452641
>>2627

>Означает ли это, что тот же метеозонд, поднимаясь вверх, "сам собой" увеличивает свою линейную скорость относительно некоей неподвижной точки наблюдения на орбите Земли вокруг Солнца, и при этом его линейная скорость относительно перпендикуляра к поверхности Земли все тот же 0?



Так и есть. Но как только ты выйдешь за пределы атмосферы, сила, увлекающая тебя за вращением Земли, пропадет и дальше бесплатно разгоняться не выйдет, линейная скорость перестанет расти, а угловая начнет уменьшаться. В итоге тебе будет казаться, что ты все еще поднимаешься строго вверх, но Земля внизу начинает все быстрее проворачиваться под тобой (на самом деле это ты отстаешь от ее вращения).

Вот если бы был способ сохранить постоянную угловую скорость и наращивать линейную, тогда да, можно бы было выйти на геостационар, просто поднимаясь вертикально. Именно так и работает космический лифт.
192 452642
>>2640
Но расстояние до центра Земли же не меняется на геостационаре (да и на Земле если стоять), значит скорость относительно центра Земли равна 0
193 452645
>>2642
То, что расстояние не меняется не значит, что скорости нет.
Привяжи к вервке что-нибудь и покрути - расстояние от центра вращения не меняется, объект статичен?
194 452646
>>2642
Это такой троллинг тупостью? Ты в курсе, что такое радиус-вектор, и что у него не только длина есть, но и направление?
195 452654
>>2646
Теперь понял, да, был не прав, прочитал определение и что такое р-в https://ru.m.wikipedia.org/wiki/Скорость
196 452674
>>2641
Спасибо, теперь понятно.

Кстати, расчет высоты ГС орбиты правильный?

(Высота орбиты + радиус земли в км) х скорость вращения на нужной широте в км/c / радиус земли = первая космическая = корень(400000 / (высота орбиты + радиус Земли))

H = (400000 х 6371^2 / V^2)^(1/3) - 6371 = 35817 км для экватора
197 452697
>>2073
чисто геометрически если прикинуть:
Земля же вокруг солнца вращается.
Мне сейчас лень выяснять, где именно находится эта Бетельгейзе.
Например, в январе онабудет освещать тёмную сторону Земли, т.е. Земля находится между Солнцем и Бетельгейзе. В этот период ночи нет. Земля освещается с двух сторон.

Далее Земля катится по своей орбитеи уже через три месяца она освещается постоянно только 3/4 времени. Ну, у неё такой получается тёмный сектор, куда не достаёт ни Солнце ни Бетельгейзе.

Через полгода(в июне-июле) Бетельгейзе для земного наблюдателя скрывается позади Солнца. Получается обычная смена дня и ночи. потом, по мере приближения Земли к той новогодней точке, ночь становится всё короче и короче и в районе НГ опять наступает постоянный день.
198 452726
Как для сайфайной истории МММММММАСКИМАЛЬНО ТВЁРДО объяснить огромную по размерам обитаемую планету с гравитацией Земли? И какого максимального размера должна быть это планета, чтобы не пересечь каких-нибудь допустимых пределов?
Изначально предполагается некая планета размером с Нептун, но, само собой, с твёрдой поверхностью. Если удастся вырастить такую твёрдоповерхностную обиташку до размеров Сатурна реалистичным насколько это вообще возможно обоснуем - будет заебись.
199 452727
>>2726
Она может быть раза в два наверное больше земли за счет состава без железа, камень раза в четыре менее плотный.
Untitled.png47 Кб, 956x624
200 452728
>>2726
Матрёшка из нескольких тысяч вложенных сферических оболочек. Каждая оболочка держит ту, что над ней, неебическими колоннами. Расстояние между соседними оболочками в несколько километров.
201 452730
>>2728
Один удар даже небольшого астероида и планета всмятку.
202 452731
>>2730
Я думаю, что если можешь такие планеты строить, то и астероиды отводить можешь.
203 452733
>>2730
Э, слыш, один удар тебе по хлеборезке и ты всмятку, дерзкий.
204 452790
>>2728
Проще уж тогда построить тупо жёсткий слой-покровало, равномерно покрывающий весь необходимый радиус. Изнутри подпирается 6-8 длинными столбами/колоннами , таким образом при размерах, допустим, близких к нептунианским, весь весь такой конструкции будет составлять массу планеты + массу столбов-подпорок + массу слоя поверхности с содержимым.
При этом даже вес можно подправить, придав внутренней настоящей планете вращение, необходимое жителям внешней стороны поверхности корки для их нужд.
image.png141 Кб, 800x450
205 452858
Поясните, допустим есть черная дыра с размером в Землю и на нее летит с огромной скоростью объект с размером в Солнце, что будет? Взрыв? Или черная дыра остановит солнце?
206 452863
>>2858
Понятие взрыв в космических масштабах можно трактовать по-разному. Еще на подлете дыра начнет поглощать вещество звезды, формируя светящийся диск на своей низкой орбите. Но если скорость приближающейся звезды достаточно велика и столкновение лобовое, то тут только гадать в духе "светящаяся материя звезды обволокет тело дыры, огромное количество материи быстро спрыгнет за горизонт, остальное ебанет мощнейшим джетом и пополнит акреционный диск".
207 452866
Что внутри чёрной дыры?
208 452867
>>2863

>Но если скорость приближающейся звезды достаточно велика и столкновение лобовое


Ну я вот про это, да. Типа представим возможную маленькую черную дыру и очень огромный объект который летит прямо на нее на максимально возможной скорости.
209 452868
>>2867
Ну, представлено тебе в кавычках. Что еще хочешь? Масса дыры с ГС в Землю уже несколько сот солнечных. Точно не скажу, считать лениво. Такой одна звезда - как зернышко скушать.
1546408428076.jpg122 Кб, 740x746
210 452870
Вокруг черной дыры вращается диск, который светится. Почему он светится? И как долго он может светиться? Бесконечно? Там какие-то процессы наверное происходят, чтобы оно светилось, но это же не может быть как вечный двигатель, который светится бесконечно?
211 452896
>>2870
Молекулы вещества ебошат на релятивистских скоростях по орбите вокруг ЧД, часть этих молекул сталкиваются друг с другом с выделением энергии и приобретением ускорения, это мы и наблюдаем как излучение. Нет, вечно светится оно не будет, потому как энергия теряется и большинство вещества падает в ЧД, а часть его улетает в космос.
212 452897
>>2870
Ладно, никто не хочет тебе отвечать, так что придется мне, раз уж начал.

>Почему он светится?


Низкая орбита вокруг ЧД подразумевает релятивистские скорости. Так как под самим горизонтом орбитальная скорость вокруг центра масс подразумевается сверхсветовой, которой быть не может, поэтому и горизонт. А все, что совсем рядышком, дабы не проваливаться, летает оче быстро. Когда дыра жрет материю, она жрет очень жадно, словно голодная обезьяна - крошки летят. Материя утекающей звезды под влиянием закритических показателей метрики и градиента гравитации, разлетается всюду как только может. Отсюда такое количество материи в акреционном диске.
А релятивистские означают что? Правильно - высокие энергии. Сталкиваясь между собой материя светит в огромном спектре радиодиапазонов, в том числе и в видимом.

>И как долго он может светиться? Бесконечно?


Нет, разумеется не бесконечно. Теоретически все вещество в диске через n10000000 лет так или иначе прореагирует - упадет под горизонт/вылетит в джете, и вообще вселенной в привычном виде настанет пиздец. Но в рамках бытовых представлений о времени, то бишь практически, можно сказать что бесконечно.
213 452899
>>2896
Братан... И почему я не рефрешнул.
214 452900
>>2897

>материя светит в огромном спектре радиодиапазонов, в том числе и в видимом.


А за счет чего свечение? Фотоны или какие частицы бесконечно будут рождаться в материи? Типа две частицы могут бесконечно друг об друга тереться и бесконечно светиться? Они же деградируют как-то.
FusionintheSun.svg.png299 Кб, 1920x2736
215 452902
>>2900
Как по-твоему происходит термоядерный синтез в теле звезды? Много много газа, по сути ионизированного водорода, читай протонов, сталкиваются на высоких скоростях в ядре звезды, потому что под действием гравитации они сильно все сдавлены и плотность реакции достаточна, чтобы "зажечь" самоподдержание. При столкновении двух частиц на определенных энергиях и удачном векторе значится преодолевается определенный барьер сил отталкивания - происходит синтез. Схемку для Солнца приложил. При синтезе выделяется энергия. Уходит квантами - фотонами, и другими частицами мелкого помола - нейтринами всех мастей и подвидов. Уход этой энергии и есть твоя деградация. Если частицы столкнулись в аккреционном диске, погасили встречную скорость, которая обеспечивала им орбиту, и рухнули нахуй под горизонт. Квант с энергией, в зависимости от вектора, либо съебался, попав в глаз мимопролеталу космонавту, либо тоже убежал под горизонт.
Просто материю дыра жрет самую разную, не только водород, а что попалось поблизости. И там уже множество тонких и плохо изученных, больше теоретических процессов происходит вроде излучения Хокинга. На самом деле это все к фундаментальным вопросам современной физики отходит, так что хочешь глубже - ищи уже сам. Можешь начать с википедии, там на самом деле все неплохо описано, но много буков.
216 452936
посоветуйте годных документалок(не старше 10 лет, чтобы какой-никакой графон) кроме очевидных с Тайсоном, ну или лекций интересных про космос. без разницы рус/англ
217 452943
>>2897
Так ведь орбиты возле гс нихуя не стабильные.
218 452944
>>2943
Ну, факт. Как-то противоречит написанному? Я просто спать уже хочу, рили не одупляю.
219 452946
>>2944
Меня смутил срок, который ты отвел на болтание материи на орбите рядом с гс.
220 452959
Есть ли у нашей галактики выраженное направление движения? Например, если построить шар с центром в нашей, расширять его и складывать векторы скоростей (или правильнее, импульсов?) всех галактик в его объёме, будет ли суммарный отличен от нуля на более-менее стабильную величину и направление?
221 452960
>>2959

> всех галактик в его объёме


Ну кроме нашей офк
222 452961
>>2959
Естественно. Все дружно ломятся к ближайшему центру масс.
223 452962
>>2961
Так в условии шар постоянно расширяем, центор масс меняться будет.
224 453008
Оставлю ссылку на вопрос в соседнем треде дабы не копировать
>>453007
Хелп!
225 453073
Аноны, а какова вообще вероятность того, что где-нибудь во Вселенной возникли такие же человеческие особи как и мы? С такими же биологическими особенностями и т.п.
226 453078
>>3073
ноль, запятая, много нулей, очень много нулей, один
227 453085
>>2936
BBC Planets

> не старше 10 лет, чтобы какой-никакой графон


Графодрочеры уже совсем охуели. Тебе документальность нужна, или визуал?
228 453086
>>3085
Визуал это важнейшая составляющая документальных фильмов.
229 453090
https://ru.wikipedia.org/wiki/Космос-2_(ракета-носитель)
Что за ТМ-185?
Вообще ебическая ракета, конечно, 6 видов жидкостей. Да еще и окислять гептильчик кислородом при живых-то азотных окислителях...
230 453091
>>3090

>гептильчик кислородом


По теоретическому УИ это весьма хорошая комбинация - 360 с на том уровне технологий кажется. Если мне не изменяет память, Глушко с ней возился очень долго, изначально хотели на таком двигателе запускать третью ступень Р-7 для лунных аппаратов, но не довели до ума.
231 453096
>>3078
Чому? Может только приблизительно такие как мы разумные существа и могут развиться (на определенном этапе, потом дальше эволюция пойдет)
232 453104
>>3086
Визуал - важнейшая составляющая только в попкорновых блокбастерах, а в документальном кино наиболее значимы достоверность приводимых фактов, подробность их изложения, короче сценарий.
233 453117
>>3104
Если тебе нужна достоверность и подробность, почему ты пейперы не читаешь?
234 453118
>>2962
Вселенная гомогенная на больших масштабах, так что центр будет более-менее в одном месте.
235 453127
>>3104
А ничего что в астрофизике за 10 лет многое поменялось?
236 453176
>>1413 (OP)
почему не в честь ученого тред назван?
237 453177
Могут ли кометы путешествовать по нескольким солнечным системам, если нет, то можно ли как то им помочь с этим?
238 453178
>>3177
Постоянно? Нет, мимопролетать? Запросто.
15468012940070.jpg657 Кб, 1400x1400
239 453179
>>2936
Очень нравятся документалки ВВС с Джимом Аль-Халили, космическая тема не везде затрагивается, но визуал и сама подача очень достойная
240 453180
>>2936
PBS Space Time
241 453187
Такой тупой вопрос: возможно ли, что существуют объекты размера Ultima Thule или больше, которые находятся ближе к Земле, чем она, и являются ещё не открытыми? Или такая возможность исключается?
242 453218
>>3187
Ну если Туле спецом искали в районе Плутона уже во время полёта Горизонтов и нашли только в 2014, то как ты думаешь?
243 453238
На Марсе сильно внешний вид и положения созвездий изменятся?
244 453239
>>3238
Вообще никак.
С земли-то этот параллакс заметен только очень чувствительными приборами, а Марс лишь в полтора раза больше гуляет вокруг солнца, чем земля.
245 453336
>>3073
Двачую вопрос. Есть ли такие учёные которые поднимали бы этот вопрос? Если да - покидайте сабж.
246 453379
>>3073
если принять факт о бесконечности вселенной, то вполне вероятно что такое где-то есть
247 453381
А расширение вселенной в масштабах солнечной системы просто пренебрежимо мало или оно к тому же ещё как-то компенисируется обратно, так что даже через триллион лет при тех же условиях ничего не распадётся?
248 453383
>>3381
Да.
249 453389
>>3218
Помогите с ответом, у меня низкий IQ ответ - да, скорее всего существуют?.
250 453447
>>3389
Да. Скорее всего, существуют.
251 453448
>>3336
Тит Лукреций Кар - "О природе вещей" ("De rerum natura"), I век до н. э.

И по природе своей настолько бездонно пространство,
Что даже молнии луч пробежать его был бы не в силах,
В долгом теченьи чреды бесконечных веков ускользая
Дальше вперёд, и никак он не смог бы приблизиться к цели.

...остаётся признать неизбежно,
Что во вселенной ещё и другие имеются земли,
Да и людей племена и также различные звери.

Est igitur natura loci spatiumque profundi,
quod neque clara suo percurrere fulmina cursu
perpetuo possint aevi labentia tractu
nec prorsum facere ut restet minus ire meando...

...necesse est confiteare
esse alios aliis terrarum in partibus orbis
et varias hominum gentis et saecla ferarum.
252 453472
>>3381
Расширение вселенной это просто когда галактики разлетаются друг от друга. Расстояния между звездами планетами спутниками и атомами не меняется.
1342535409802.jpg47 Кб, 500x500
253 453485
Антон, а существует какой-нить онлайн-телескоп бесплатный? Хочу сидеть уютно под пледиком и смотреть в небо через телескоп. Из ит поссибл?
254 453489
>>3485
Гугли онлайн-планетарий, качай Celestia и подобные, и Space Engine зацени.
255 453490
>>1745
Потому что заяц игрушечный.
256 453491
>>3489
Селестия уже давно не обновляется. Это минус. но для новичка сойдет.
257 453492
>>3491
Охуеть. С 2011. Примерно тогда же когда по запросу "Селестия" перестал выдаваться планетарий. Совпадение?

Тащемта есть не менее бесплатный Stellarium.
15469393247940.png1,2 Мб, 700x875
258 453519
Допустим что Солнце было бы первой зародившейся звездой в нашей вселенной миллиарды лет назад. Там бы появилась Земля и аноны, ну и остальная солнечная система. Что бы мы видели на небе в тот период. Изменились бы как то наши представления о космосе?
omygif.gif774 Кб, 320x270
259 453521
>>3519
В тот период во вселенной не было элементов химических кроме водорода, гелия, лития. Для органики и жизни элементы начали появляться когда те первые звезды насинтезируют в себе их и взорвутся, только из их ошметков будут звезды с твердыми планетами и жизнями
260 453523
>>3521
какой ты скучный. А если мы попаданцы?
261 453533
Бля, челы срочно надо видос(ссылочку). Там сидел мужик, клацал по гугл картам и говорил о мифе численности землян, типа -" вот посмотрите, на остановке 3 человека если бы нас было 7 миллиардов то тут было бы 500"
262 453542
>>3127
Да что ты? Может, орбитальная механика поменялась? Или случился небывалый прорыв в космологии, который перевернул все представления о вселенной?
Нихрена в "астрофизике" не поменялось, просто накопилось немного больше данных, было совершено несколько интересных открытий, выдвинуто некоторое количество гипотез... В общем все идет своим чередом.
А бибисишные документалки как были актуальны на момент их выхода, так и не сильно потеряли в актуальности на сегодняшний день. Если же тот, кто изначально интересовался этим вопросом, уже перерос данный там уровень знаний, и его интересуют самые последние открытия, то про это документалок скорее всего еще просто не сняли. Так-то.
263 453587
>>3542
Ну вот как минимум пересмотрели источник большей части тяжелых элементов Вселенной со сверхновых на слияние нейтронок, вполне себе фундаментальная штука, причём как раз уровня научпоп кинца.
264 453588
>>3587

>слияние нейтронок


+наблюдения гравитационноволновых всплесков от них
265 453602
>>3587
Ты хочешь сказать что эти модели коллапса сверхновых с массовым выплеском железа и синтеза тяжелых элементов из-за колоссальных температур и скоростей теперь хуйня?
266 453603
>>3602
У них нехватало энергии на золото платину и прочие ураны.
267 453604
>>3602
Не хуйня, но, насколько я понимаю, сейчас пришли к выводу, что этого маловато и для синтеза всего, что тяжелее железа, этого сильно не хватает. А вот если добавить слияние нейтронных звезд, которое тоже будет их создавать - хватает. Но тут я слабовато ориентируюсь в теме, на самом деле, надо еще самому почитать и разобраться как следует.
image.png114 Кб, 293x172
268 453605
>>3603
Там же суть была, что прорывающиеся джеты из ядра (в основном полюсные, но вообще повсюду) из-за давления, ну вы знаете как коллапс рисовали, это дохуища материи вроде железа и до него и оно все проходило через падающее вещество, там энергии были колоссальными настолько что даже несколько трансмутаций и столкновений за мизерное время успевало случиться.

Пожалуйста, расскажите новые веяния, где почитать (а если уже и сняли в каком-нибудь PBS Space Time или MIT - дайте ссыль плиз).

>>3604
Мне очень нравится цитата которую приводил Рэндел Манро:
The physicist who mentioned this problem to me told me his rule of thumb for estimating supernova-related numbers: However big you think supernovae are, they're bigger than that.
Это из статейки про то, что сверхновая на 1а.е. тебя убъет даже нейтринами.
269 453608
>>3605
Вообще я вот этот обзор хочу почитать (хотя бы в общих чертах понятно будет что к чему), но пока руки не дошли: https://arxiv.org/abs/1901.01410
270 453609
>>3533
Пукнул
271 453628
>>1768

>в космос


И упасть обратно. В космос это не про высоту. А про скорость.
272 453630
А можно ли запустить объект на другой континент (9000км) с помощью пушки?

Спрашиваю для друга.
Ким.
1497245037163926158.jpg207 Кб, 1280x720
273 453631
>>3630
Можно. Только пушка должна быть многокаморная, невзъебенных размеров а в СК найдут наконец нефть и нехватку демократии
r3223r23.jpg73 Кб, 1940x868
274 453635
>>2897

>вылетит в джете


А эти ваши джеты придают ЧД ускорение?
275 453638
>>3635
Нет, они с обоих полюсов симметрично ебашат.
276 453642
>>3638
Хуй-на, а откуда тогда огромные скорости у некоторых черных дыр? Сразу после коллапса выбросы как раз дохуя асимметричные, это потом, когда коллапсар доедает родительскую материю и начинает лакомиться залетной, они становятся симметричными!
277 453724
>>3642
Оттуда же откуда огромные скорости и не у черных дыр.
У ЧД механизм коллапса обычно симметричен.
Дай-ка инфу об обратном, будь так добр.
279 453779
1) Супер далекие маленькие объекты солнечной системы в самые современные мега-телескопы на орбитате или с адаптивной оптикой мы видим как очень тусклые движующиеся точки без намека на подробности.
Почему ученые не сделают супер-ёба-сверхмощный сверхнаправленный узкий лазер за миллиарды долларов с энергией с томнцю бомбу, чтобы направлять его в сторону очень далеких объектов типа Плутона, Эриды, и прочих анонов пояса Койпера, чтобы доосвещать их светом-вспышкой чтобы они отражали обратно больше света и мы могли нормально узнать как они выглядят?
2) Далекие обьекты в том числе аноны-плутониоиды и далекие экзопланеты, звезды с самым большим увеличением мы видим как размазанные неуравновешанные пятна похожие на амебу. Собственно почему так? В идеале чем больше телескоп экспонирует обьект - тем четче он должен становиться, причес без погрешностей, как будто кто-то снял на 4й айфон ночное небо. Почему все самые ужаленные объекты на самых современных телескопах оставляют артефакты, а не идеально представляют собой то что они есть на самом деле.

3) Вдруг ученые сделали неправильный вывод из постоянного красного смещения всех галактик вокруг, что типа они ускоряются тем быстрее чем больше расстояние до них. И как следствие неправильно вывели ТБВ. Вдруг красное смещение вызвано другим процессом и законом, который мы кще не знаем. Следовательно основная теория астрономии и понимание вселенной идет к хуям.
280 453780
>>3779
1) и 2): Проблема не в тусклости объектов, тусклость как раз компенсируется длительностью экспозиции, проблема в разрешающей способности, увеличить которую можно только увеличением диаметра зеркала.

>В идеале чем больше телескоп экспонирует обьект - тем четче он должен становиться



Ярче — да, четче — нет. Если угловой диаметр объекта меньше максимального углового разрешения телескопа, хоть усмотрись, никаких подробностей не будет никогда. Законы физики не позволяют.

3) Может, это ты к хуям пойдешь?
281 453781
282 453782
>>3779
3) Космология основана не только на красном смещении галактик, есть дохуя других ключевых наблюдательных фактов, начиная с реликтового микроволнового фона и его свойств. Есть какие-то наблюдения, трактовка которых плохо согласуется с другими, но общий смысл космологии Большого взрыва они не затрагивают.
283 453811
Насколько реально, что НАСА в ближайшие годы одобрит миссию На Титан со спускаемым модулем? Или это из разряда фантастики?
284 453813
Технари, поясните за пару вопросов:
1. Какой формы может быть вселенная?
Если большинство объектов во вселенной стремится к шарообразной форме, даже начало всего, то есть сингулярность теоретически тоже сферическая я так понимаю, то логично, что и форма вселенной тоже сфера?
Когда сингулярность ёбнула, образуя вселенную, то сила взрыва должна распределяется равномерно и со всех сторон одинаково, то есть получается, что вселенная - это расширяющиеся шар в нигде? Логично или нет?
2. Что всё таки произойдёт со временем внутри вселенной? Ведь всё находится в движении.
Галактики движутся, а также во вселенной есть огромные скопления галактик типа Аттрактора и Шепли. Так вот, насколько такое возможно, что все скопления в итоге начнут притягивать к себе скопления меньше, пока в итоге во вселенной не останется одно единственное скопление? Это же будет очень массивная хрень, возможно ли, что это самое скопление из-за очень большой массы в итоге опять превратится в сингулярность сжав всю вселенную, пространство и время в одну точку? Ведь всё стремится к шару, то есть в одну точку. Не сильно глупая мысль?
285 453814
>>3813
1 скорее всего да, сфера
2 нет, так как расширение вселенной на больших дистанциях сильнее гравитации
286 453815
>>3814

> 2 нет, так как расширение вселенной на больших дистанциях сильнее гравитации


Ок. Тогда ещё последний вопрос. Вселенная ведь не может вечно расширяться. Что начнёт происходить, когда будет предел? Возможно ли такое, что оно, ну скажем так рваться начнёт, лол? Я к тому, что пространство, материя, тёмная энергия я хз, что тянется, но растягиваттся оно же всё время не может же, бесконечно.
287 453816
>>3815
хуй знает Современная наука не дает ответ на этот вопрос
288 453821
>>3815

>растягиваться оно же всё время не может же, бесконечно


В рамках общей теории относительности - да, может. Но есть серьезные основания полагать, что на при охулиардолетнем возрасте вселенной должны становиться существенными квантовогравитационные эффекты, которые хуй знает как посчитать - не построено квантовой теории гравитации. Может, пространство продолжит расширяться, а материя пойдет по пизде, может, наебнется и то, и другое.
289 453824
>>1736
Я бы взял большой камень, разрезал бы его, вырезал бы середину и поместил бы туда куб. Хорошенько загриммировав, я бы поместил этот камень в кольцо Сатурна.
Запомнил бы сам камень соответственно, а так же рассчитал бы траекторию движения, чтобы без проблем в будущем его найти. Вот и всё, куб в камне летит среди миллионов других камней по орбите.
И так, местонахождение знаю только я, все расчёты у меня в голове. Я буду всегда носить с собой ампулу с цианидом. Если кто-то захочет выпытать у меня местонахождение, я себя убью. Я ж надеюсь ты не представляешь 2118 год такой, что людишки смогут память извлекать из трупов?
Если вдруг каким-то раком какой-то долбоёб поймёт, что камень где-то в кольце Сатурна, то узнать в каком камне именно будет практически невозможно. С Земли каждый камень из миллионов сканировать невозможно и в 2118 тоже не будет возможным, скорее всего. Это надо обязательно лететь к кольцу. Нельзя построить робота который бы сам осматривал каждый камень и передовал инфу на землю. Любой робот будет ломаться и кончаться топливо. Каждый раз посылать дозоправщик с топливом и людей для ремонта крайне затратно. Легче будет просто забирать подозрительные камни с кольца на Землю и осматривать на Земле. Шанс найти именно этот камень 1/1 000 000 примерно, я хз сколько там камней в кольце. Время найти куб увеличивается на тысячи лет. А за тысячу лет на земле может произойти всё, что угодно: эпидемия, вирусы, нехватка ресурсов и всякая подобная ебала. Так что логично, что абсолютно все люди на планете не могут заниматься и финансировать операцию по поиску куба.
Как по мне, то шанс найти очень мал.
290 453829
>>1736

>Вопрос-викторина для людей из прошлого. Вы должны предложить особое место в Солнечной системе, где можно спрятать объект кубической формы с гранями 10x10x10 метров, который никто и никогда не должен обнаружить и затем не должен достать этот объект.


>


Прятать надо на самом видно месте- на Земляшке, в фундамент какой-нибудь строящейся ГЭС или АЭС.
291 453830
>>1736
На высоте больше двух тысяч метров над уровнем моря в Йеллоустонском национальном парке ЕТПОЧЯ. А если серьезно, то где-нибудь на дне подледного океана Европы, скажем (если там есть тектонически спокойные места).
>>3824

>рассчитал бы траекторию движения


Идея красивая, но сразу фейл - траектории там хаотические, и нихуя ничего рассчитать на длительное время не выйдет, плюс есть шанс наебнуться в случайном столкновении.
292 453848
>>1736
В Каабу запихну.
293 453850
>>1736

>Против вас по сложности выступает вся человеческая цивилизация уровня 2118 года


То есть из инструментов у них палки и камни? Ну брошу на дно морское где-нибудь.
294 453851
>>3850

> То есть из инструментов у них палки и камни? Ну брошу на дно морское где-нибудь.


Он же написал >>1787

> нужно закинуть не в случайную орбиту, а в определенную, где ты при желании мог бы его обнаружить


То есть куда нибудь рандомно нельзя.
295 453853
>>3851
Предлагаю сагать того уёбка вместо ответов. Наверняка автор говнофантастики про жопаданцев пытается на бесплатную консультацию добрых спейсачеров развести.
296 453867
>>3853
Лол, хотя очень даже вероятно.
297 453876
>>1736
Откопал бы гроб своей бабушки и засунул туда куб, закопал обратно. Некультурно же чужие могилы рыть!

Распечатал бы копию куба на 3д-принтере, покорно отдав копию. Пока вся человеческая цивилизация ослеплена невероятным успехом, оригинальный куб лежит под моей кроватью и кучей одежды.

Выехал бы в ближайшее Кукуево-Залупово и сбросил куб в неприметный деревенский туалет.

Ну как, я справился?
298 453893
>>1736

>Вопрос-викторина для людей из прошлого. Вы должны предложить особое место в Солнечной системе, где можно спрятать объект кубической формы с гранями 10x10x10 метров, который никто и никогда не должен обнаружить и затем не должен достать этот объект.


>Против вас по сложности выступает вся человеческая цивилизация уровня 2118 года, так что добраться в некие далекие отдаленные места Солнечной системы они могут. Куда бы вы спрятали КУБ, так чтобы его не нашли, а если даже найдут - не смогли достать, но так чтобы он не был полностью утерян. Это исключает ядра звезд и планет гигантов разумеется.


Границы Солнечной системы определяются сферой Хилла. Это, на минуточку, сфера радиусом световой год.
Берём облако Оорта, в котором миллиарды миллиардов кусочков ледяного говна. Заливаем твой куб льдом в 100-метровый ледяной шарик и суём в рандомное место среди бесчисленного множества таких же шариков.
Заебутся искать, хоть человечество 2118 года, хоть 2218. Даже со сверхсветовыми двигателями заебутся. Ебанёшься прочёсывать всё это пространство.
299 453895
>>3533
Вот, но это бред https://youtu.be/k6Y2nTXkUmQ
300 453897
>>3876
Таких болших туалетов нет
301 453898
>>3893

> Границы Солнечной системы определяются сферой Хилла. Это, на минуточку, сфера радиусом световой год.


Значит то, что вояджер покинул солнечную систему - это попсовый развод для впечатлительных невежд?
302 453901
>>3897
Упс, писал о кубе 10х10х10 сантиметров
303 453904
>>3895
>>3895
Спасибо
304 453914
>>3898
>>3898

>Значит то, что вояджер покинул солнечную систему - это попсовый развод для впечатлительных невежд?


Именно. Вояджер всего лишь долетел до границы гелиосферы - область, где заполнение пространства частицами становится идентично межзвёздной среде. Это примерно 120а.е. Но дальше ещё много всякого говна. Например, карликовая планета Седна 500-900а.е. и дохуя всяких долгопериодических комет с огромным афелием. Вплоть до светового года и ещё больше. Афелий у таких комет вычисляется лишь приблизительно, и у некоторых экземпляров расчётное значение превышает расстояние до Проксимы Центавры - здесь подразумевается, что либо в их траекторию что-то ещё вносит возмущение в пределах сферы Хилла, либо что существуют межзвёздные кометы, шлюхающиеся сразу вокруг нескольких звёзд. Орбитальная скорость этих объектов вблизи афелия околонулевая, периоды исчисляются миллионами лет, поэтому одних только неизвестных человечеству долгопериодических комет в облаке Оорта должно быть просто дохуя, а объектов с более равномерными траекториями ещё больше. Наблюдать их в телескоп при этом крайне проблематично - задача на несколько порядков сложнее фотографирования этих ваших Ультима Туле. Там этих ёбаных кубов сколько угодно можно спрятать.
305 453927
Пру в ми вронг.Я тут придумал охуенную идею как сэкономить на межпланетных полетах.Суть такова.С помощью телескопа ищем пролетающие мимо земли кометы.Запускаем ракету в точку рандеву.В момент когда комета пролетает мимо космического корабля цепляем ее с помощью гарпунов.Комета тянет наш корабль сообщая ему нужную скорость.Когда скорость будет достигнута отцепляемся от кометы и корректируем орбиту.
306 453928
>>3927
Чтобы зацепиться за комету, кораблю придется практически уравнять с ней скорость, иначе любой трос порвет. Это почти как высадка на комету. А если корабль движется со скоростью кометы и в том же направлении, то нахрена ему сама комета? Он и без нее будет двигаться не хуже.
sage 307 453933
>>3904
Иди нахуй, жопаданцер.
308 453942
>>3928
Чтобы трос не порвался его нужно делать из материала для космических лифтов.Алсо можно сделать катушку с регулятором натяжения как в удочке.Самое сложно это как зацепиться за комету.Нужен гарпун особои формы чтобы мгновенно фиксировался.
309 453949
>>3942
Во-первых, для космических лифтов материала именно что нет, даже метр троса не сделать с нужными характеристиками. Возможно, когда-нибудь в будущем будет такой чудо-материал, но сейчас такой трос невозможен.

Во-вторых, даже с волшебным гарпуном и идеальным регулятором все равно хуйня выйдет, потому что остаются несколько более фундаментальных проблем:

Причина 1: для торможения с космических скоростей и более-менее приемлемых ускорений потребуются сотни километров троса. Допустим, скорость кометы относительно нашего корабля 5 км/с, а аппарат и трос могут выдержать 5g длительной перегрузки, тогда торможение займет 100 секунд, а длина размотанного троса составит 250 км.

Сколько будут весить 250 километров троса, способного выдержать пятикратный вес аппарата, точно не посчитаешь, но ясно одно — дохуя. И увеличение тормозящего ускорения для уменьшения требуемой длины троса особого выигрыша, ведь с ростом ускорения на трос будет приходится большая нагрузка, а значит, и сам трос должен быть толще и тяжелее. Плюс это еще мы по самому минимуму посчитали, а в реальную миссию веревку придется брать с запасом и по длине и по прочности.

Причина 2, еще важнее: кинетическая энергия аппарата вовсе не исчезнет бесследно, а практически вся перейдет в тепло в месте трения тормоза о трос. Т.е. для наших 5 км/с на каждый килограмм массы аппарата на катушке выделится 12,5 мегаджоулей тепла (на самом деле еще больше, так как в начале вместе с аппаратом будет тормозиться еще и масса неразмотанного троса).

12,5 МДж это примерный эквивалент трех килограмм тротила, т.е. в твоей катушке фактически разорвется бомба, весящая в несколько раз больше самого корабля, только не сразу, а за несколько десятков секунд. Куда ты собираешься отвести такое колоссальное количество энергии, я вообще без понятия, в космосе с отводом тепла проблемы. У обычных двигателей тепло уносится с рабочим телом, но для тебя такой вариант недоступен.
310 453951
>>3949
Пока писал, подумал, куда можно отвести часть тепла и заодно сократить длину троса, правда, идея больше подходит для омского КБ.

Подключаем к катушке генератор, а генератором питаем ЭРД, только не дохлый ионник, а что-нибудь попроще и с большей тягой: аркджет или резистоджет, короче, термоэлектрический двигатель — как ЯРД, только нагрев электричеством, а не реактором. В итоге и троса меньше надо, так как торможение быстрее, и тепло отводится, правда, все равно нужен ракетный двигатель и рабочее тело к нему, но в меньшем объеме.
14936538084880.jpg68 Кб, 564x557
311 453960
Поясните за млечный путь. В этой галактике кроме планет (Меркурий, Венера, Земля, Марс, Юпитер, Сатурн, Уран, Нептун) есть ещё какие нибудь планеты? Кстати Солнце в этой галактике тоже только одно?
1547184872706.jpg47 Кб, 600x455
312 453963
>>3960
Ты так троллишь?

> (Меркурий, Венера, Земля, Марс, Юпитер, Сатурн, Уран, Нептун) есть ещё какие нибудь планеты?


В нашей галактике от 800 миллиардов до 3,2 триллионов планет

> Кстати Солнце в этой галактике тоже только одно?


В нашей галактике где-то 150/250 миллиардов звёзд.
Самих же галактик во вселенной сотни миллиардов в каждой из которых входят сотни миллиардов звёзд и планет.
313 453967
>>3963

>В нашей галактике от 800 миллиардов до 3,2 триллионов планет


У них названия есть?
314 453968
Какие нахуй Бенну и Ультимы Тхуле? Нахуя наса запускает КА ко всякой параше ну УТ еще хуй с ним, там по пути? Где мои миссии на Европы с Титанами и к прочим Нептуняшам с Венерами?
315 453970
>>3968
Хуле ебало скорчил? Ближе к тридцатым годам будут кроме нетуновой миссии, ее не будет
316 453971
>>3967
Есть, большинство других планет носят названия порядкого номера, чтобы было легче ориентироваться где какая, например: HD 40307 g, HD 85512 b, Kepler-22 b, Kepler-62 f и т.п.
asteroid-pic905-895x505-17419.jpg63 Кб, 895x505
317 453972
Анон, насколько вероятен/выполним сценарий?

Какая-нибудь космическая держава, например, Китай придумывает хитрый план. Находит в космосе какой-нибудь мимоастероид, пусть тот же Апофис, или какой побольше, и отправляет к нему какой-нибудь двигатель, который будет тихонько себе коптить год-другой или сколько понадобится, чтобы подкорректировать орбиту. И в итоге уронит этот астероид, скажем, на США.
15396114410960.jpg10 Кб, 221x228
318 453973
>>3970

>ее не будет


Они там охуели?
319 453976
>>3960
Ты путаешь Солнечную систему с Галактикой. Солнечная система - это Солнце плюс всякий хлам на орбитах вокруг него. Солнце - одна из звезд в Галактике. Про Галактику тебе всё уже написали. В Солнечной системе, кроме известных звезд (Солнце) и планет (ты их перечислил) есть дохуя более мелких тел. Может быть и еще одна крупная планета, но она гораздо дальше от Солнца, чем другие планеты - иначе ее бы давно нашли. Шансов, что вокруг Солнца вращается еще одна звезда, практически нет - чтобы ее до сих пор не заметили, она должна быть гораздо меньше Солнца и настолько далеко от Солнца, что ее, скорее всего, унесло бы во время сближения Солнечной системы с другими звездами Галактики.
320 453977
>>3972
Никак невыполним. Нет таких двигателей, носителей, астероидов, орбит, топлив, средств обеспечения секретности, средств наведения и задач. Даже в фантастических проектах на сто лет вперёд.
Астероид наносящий невосполнимый урон США наносит его всем в том же объёме, но на несколько суток позже.
321 453979
>>3977
Но в планах терраформирования же есть пункты про бомбардировку ледяными астероидами. Значит как-то это себе представляют они
322 453980
Аноны, ведь наша планета населена разумной формой жизни. А вообще как нибудь со стороны это заметно? То есть если из далека наблюдать за землёй, особые электромагнитные какие нибудь излучения исходят от планеты, показывая, что на ней цивилизация есть?
И ещё, мы же юзаем оптоволокно в последнее время, я слышал, что оно вроде не фонит ничем, то есть никаких заметных волн. Это правда? Так вот, возможно мы ни о ком не знаем, потому что инопланетяни давно юзают что-то похожее на оптоволокно?
323 454001
>>3980
Если в пределах 300-1000 световых лет есть цивилизация, способная найти нашу Землю просто как планету, с большой вероятностью, когда до них дойдут радиоволны, которые в большом количестве излучались начиная с 40-х 50-х годов (в основном мощные радары), они явно заметят избыток радиоизлучения. Возможно даже найдут в шуме что-то интересное. Сейчас общая интенсивность раадиоизлучения падает (оптика, да, плюс помирающие радиостанции, особенно всякие там длинноволновые), при этом оно становится всё более шумоподобным из-за сжатия цифрового сигнала. Кроме того, военные радары тоже со временем удет всё сложнее опознать, т.к. идёт отказ от испульсов через регулярные предсказуемые интервалы. Так что превышение в радиодиапазоне есть и будет, но оно будет снижаться и сложнее опознаваться как искусственное.
324 454005
>>4001

>Если в пределах 300-1000 световых лет есть цивилизация, способная найти нашу Землю просто как планету, с большой вероятностью, когда до них дойдут радиоволны


Так интенсивность же ничтожная будет на таком расстоянии, не?
325 454006
>>4005
Ну были расчеты где-то, что тарелка в Аресибо способна обнаружить нашу цивилизацию с ~300 светолет, почему бы у инопланетян и не быть тарелочкам получше.
326 454007
>>4006
А. Ну ок
327 454009
>>3811
Бамп
328 454010
>>4009
Они вместо годноты на всякие говнокамни нинужные летают.
329 454012
>>3811
Дашь денег, миллиарда так три долларов одобрят.
Не дашь, будет в общем конкурсе.
330 454021
>>3811
g: Dragonfly Titan
331 454028
>>4012
Будь я Безосом - задонатил бы. А чому НАСА не использует краудфандинг например? Я думаю напшлось бы огромное количество людей, готовых пожертвовать на такое
332 454033
>>4028
Собрать с мимокроков лям далларов на инди-поделку реально, а собрать с мимокроков пару миллиардов как-то маловероятно.
333 454150
>>3968

>с Венерами?


А хуле на ней делать? Там же пиздец.
334 454165
>>4033
Ну например ТЕСС стоил всего пару сотен млн, из которых более половины присвоил Муск, запустив крошечный 500кг аппарат по цене чуть ли не в два раза выше многотонных спутников.
335 454313
Если с Марса стартует ракета, со скоростью света, к Земле то она придет тогда же когда дойдет свет. То-есть, для Земли прибудет
мгновенно?
А если ракета стартует с Земли, обогнет Марс и вернется, для наблюдателя с Земли будет казаться что весь её полет занял столько же времени сколько свет с Земли дойдет до Марса?
То-есть курьер доставляющий сообщение на бумажке со световой скоростью и возвращающийся с ответом, сделает это вдвое быстрее радиоэлектронной аппаратуры или я что-то не догоняю?
336 454314
>>4313

>То-есть, для Земли прибудет


>мгновенно?


Нет, минут пятнадцать со времени старта. Старт мы не заметим.
Вот для ракеты времени не пройдет.

>А если ракета стартует с Земли, обогнет Марс и вернется, для наблюдателя с Земли будет казаться что весь её полет занял столько же времени сколько свет с Земли дойдет до Марса?


Да.
Нет, столько же времени, сколько и аппаратуре надо, свет не обгонишь.
Время идти будет иначе для курьера.
Его хоть на другой край вселенной пошли - если со скоростью света сделаешь, для него времени не пройдет, сама же вселенная за это время состарится на миллиарды лет (смотря как далеко лететь).
337 454365
Как перестать думать о том, что за пределами видимой области вселенной?

Я скоро кукухой поеду. Человеческое сознание не может принять это, это сумасшествие.
338 454370
>>4365
Так не думай.
Что за пределами твоих 8 гб ОЗУ на компе? Что было в твоем сознании до твоего рождения?
Понять "НИЧЕГО" нельзя, это можно лишь принять.
И, сука, можно точно так же принимать математические концепты вроде сингулярности и других, понять ты можешь не все.
339 454385
>>4370

>Что за пределами твоих 8 гб ОЗУ на компе?



Блядь, ОЗУ на заводе сделали.
340 454390
>>4365
То же что за пределами обычного горизонта - все та же обычная хрень что тебя окружает.
341 454405
Вокруг земли около 7 звезд на расстоянии 4-7 световых лет на парочке есть экзопланеты.

Можно ли 10 ступенями разогнать нано сат до 5% от скорости света, что бы создать межзвездную экспедицию в 100 лет?
342 454414
Что случится с нами если планета моментально остановится? По идее же нас инерция должна распластать будет или как?
343 454415
>>4414
Остановится относительно чего?
344 454416
>>4415
Вокруг своей оси*
345 454419
>>4150
Изучать, епта.
346 454431
>>4385
ЗА пределами ОЗУ у тебя как память реализована, додик?
2018-02-03-051128.jpg156 Кб, 944x613
347 454475
Почему японцы так любят твердотопливные ракеты?
348 454478
>>4475
Дешево и сердито?
349 454479
>>4475
Нет.
350 454522
>>4405
5% от скорости света - 15 000 км/с. Химическими ракетами и 150 то нельзя. Даже ионниками не получится. Нужно что-то вроде "Ориона" или лазерами с Земли разгонять.
351 454524
>>4522
А сколько можно за счет гравитационных асисстов от планет выжать?
352 454530
>>4524
Тридцать-сорок. Сопоставимо со скоростями самих планет, в общем.
353 454536
А эту ебанину можно построить на даче?

Ну то есть если собрать сферку из кусков стекл, да стекла будут хуевыми и сферка будет не ровная, но все равно ведь будет шин?

Как по мне - не сложно. метров так 10 в диаметре из обычных зеркал на тарелке из метал профиля,

Можно ли будет рассматривать гейзеры на спутниках Юпитера? или с увлечением диаметра нужно по экспоненте дрочить на точность в микронах и все будет максимум мыло мыльное?
354 454539
>>4536

>А эту ебанину можно построить на даче?


Без лазера только, что делает ее обычным оптическим телескопом.
Можно, никто не запретит. На сверхмощный натриевый лазер надо разрешение и нелетная зона.

>Ну то есть если собрать сферку из кусков стекл, да стекла будут хуевыми и сферка будет не ровная, но все равно ведь будет шин?


Шина не будет. Зеркала модульные из-за того, что они микроконтроллерами подстраиваются чтобы исправить искажения атмосферы.

>Как по мне - не сложно. метров так 10 в диаметре из обычных зеркал на тарелке из метал профиля,


Выйдет хуже, чем обычный рефлектор за 100к. В оптике важна точность обработки зеркал, ты на покупных зеркалах такого не добъешься.

>Можно ли будет рассматривать гейзеры на спутниках Юпитера? или с увлечением диаметра нужно по экспоненте дрочить на точность в микронах и все будет максимум мыло мыльное?


Мыло мыльное.
За конкретными цифрами зайди в телескопотред, тебе расскажут какая апертура и прочие умные слова нужны для того, чтобы видеть то-то или иное: >>429065 (OP)
355 454559
А наблюдается ли притяжение луны прямо визуально кроме как приливами?
Не было ли стенда в вакуумной камере где идеально отлитый пол и ровный шарик на нем катается притягиваясь к луне?
356 454566
>>4559
Можно смотреть в телескоп за отклонением метеоритов.
357 454573
>>4566
Я имел в виду чтобы прям любому быдлану показать можно было - смотри - это луна, у тебя есть другие обьяснения?
358 454601
Если бы у китайцев хватило денег, мог ли Шугуан полететь?
https://ru.wikipedia.org/wiki/Шугуан
359 454661
>>4601
Нет, так как кадры были репрессированы в ходе "культурной революции". Гораздо интереснее следующее:
https://en.wikipedia.org/wiki/Fanhui_Shi_Weixing#Manned_flights
WTF?
360 454669
>>4539
А вот еще.

К примеру, нацелились мы на некую планету обычной камерой смартфона, и дает она нам пятно площадью 16х16 пикселей.

Возможно ли как-то чисто программными методами отфильтровать со временем шумы от засветки, атмосферы, тепловые - и, обработав последовательность из 100500 мыльных пятен, получить в итоге уже 256х256, с идеальной цветопередачей, без шумов?

То же самое с туманностями.
Oceanplanet1.png526 Кб, 1024x768
Дно у планет-океанов 361 454671
Сап, спейсач
Да, я знаю что есть тред про экзопланеы но он немного сдох.
В в теории могут существовать планеты целиком покрытые водой, на дне которых вместо горных пород располагается лёд под высоким давлением. И из-за этого существование жизни там вряд ли возможно, но если бы на на дне находился слой осадочных пород, то организмы смогли бы получать оттуда минералы и пр.
Так может ли он появится из-за падения метеорита космической пыли и тд и тп. И если да то какой толщины?
362 454672
>>4671
Ты должен запретить себе думать в рамках четкой границы лёд-жидкость. Такого не бывает, на поверхности льда всегда есть определенный налёт веществ, которые не являются водой. Конечно же там будет слой минерального вещества, правда зависит от глубины, которая может варьироваться. Опять же планеты это не идеальные в вакууме, на такой планете будут высшие точки и низшие точки. Если давление большое, то из мантии будет выделяться дерьмо, как у нас черные курильщики выделяют. Оно будет растворяться в воде, вот тебе всякие минералы, соли.То есть планета не будет на 100% из воды состоять, будь с собой честен, в центре будет железно-кремниевое ядро в любом случае, оно будет периодически бурлить, булькать, эндогеникой выделять газы с частицами минералов на более высокие уровни, даже если им придется проходить не через кору из твёрдых пород, а через лёд под большим давлением. Будут черные курильщики, будут подводные вулканы, подводные извержения в виде гигантских пузырей газа, которые пришли откуда-то из центра планеты и принесли частицы всего по-немногу, что оказалось на дне со времен аккреции, всё это будет попадать в воду выше и разноситься течениями, другими конвективными потоками. Что-то из раствора будет оседать на "дне" из льда. Что-то будет подниматься на поверхность или в виде взвеси плавать, что-то всплывет и будет булькать в атмосферу. Но сможет ли в этом говне забродить жизнь опять же зависит от глубины, давления, температуры, доступа к другим источникам энергии. В любом случае жизнь там может быть примитивная, как нам сейчас кажется.
363 454674
>>4672

>может быть только примитивная


фикс
364 454679
>>4028

>задонатил бы


Вот поэтому ты и не он, левачок.
365 454680
>>4672
Может оказаться так, что подобные планеты и есть главные инкубаторы жизни во вселенной, а планеты типа Земли скорее исключения из правил. Как никак выживать в воде намного проще, меньше влияния от радиации, перепадов температур, падений каменюк с неба, гамма-вспышек и так далее. Можно вращаться вокруг охуевшего красного карлика в приливном захвате, - течения обеспечат более менее равномерный нагрев, а толстый слой воды надежно защитит от солнечных вспышек.
366 454681
>>4680
Там с такими планетами есть подводные камни. в смысле проблемы, а не буквально подводные камни. типа может происходить такая штука как разбушевавшийся парниковый эффект, который превратит планету в чайник и поверхность воды будет кипеть и не будет поверхности вообще, а будет некий парообразный горячий конденсат у поверхности, похожий на туман, только горячий туман-пар, переходящий в жидкость очень плавно.
Из космоса планета будет выглядеть как Венера, только облака будут белые и не будет видно поверхности из-за бесконечных белых облаков. Гребанный чайник, чо с него взять. Охуевший красный карлик все равно будет пиздить такую планету и сбивать с неё атмосферу на протяжении миллиардов лет.
Про приливный захват я не знаю конечно, зависит от массы, если она большая, то планета даже в приливном захвате будет вообще поебать горячая и не будет льда на другой неосвещенной стороне. Есть же Нептун, который выделяет тепла больше чем получает, также и это говно будет остывать очень долго, а еще зависит от звезды конечно.
367 454682
>>4680
Суша на Земле это скорее как возможность для развития, вода же никуда не делась.
368 454705
>>4669
Такой метод применяется для астрофото планет Солнечной системы, стакаются кадры из видео, программно обрабатываются и получается лучше чем с одного кадра. Но разница там на порядок меньше чем у тебя, плюс так фильтруются в основном тепловые шумы от атмосферы. Кроме того, поскольку это видео, то тебе нужна короткая выдержка каждого кадра и, как следствие, большая яркость объекта, что сразу отметает всё за пределами СС из области применимости.
369 454713
Господа шароверы, откуда на обращённой к Земле стороне Луны метеоритные кратеры? Ведь Земля прикрывает Луну от всяческих космических объектов, если там и быть кратерам, то только на обратной стороне а поскольку Луна, как и Земля, плоская, то никакой обратной стороны не существует
370 454717
>>4713

>Земля прикрывает Луну


Какое же ты двумерный, пиздец просто.
371 454720
>>4713

>поскольку Луна, как и Земля, плоская, то никакой обратной стороны не существует


У плоскости нет обратной стороны плоскости?
Вы там совсем ебанулись?
372 454725
Какой мощности и размера нужен телескоп, чтобы увидеть планеты в других звёздных системах? Именно увидеть
373 454726
>>4725
Метров 10 апертуры хватит.
374 454750
>>4720

>У плоскости нет обратной стороны плоскости?


А как выглядит обратная сторона плоскости? Является ли она зеркальным отражением лицевой стороны?
375 454804
>>4725
Мы в своей системе планету-то увидеть не можем, а ты на другие замахиваешься.
image.png99 Кб, 1089x909
376 454805
>>4713

>Земля прикрывает Луну от всяческих космических объектов


Это как она прикрывает так, чет не понимаю? У тебя космические объекты по рельсам ходят? Но это даже в 2д мире не так
377 454807
>>4805
Он из 1д мира просто.
378 454809
>>4805
Между тем на обратной стороне луны кратеров больше чем на обращенной к нам.
379 454814
>>4809
Только потому, что на ближней стороне древнейшие кратеры затерты лунными морями.
380 454816
Технически возможно ли сегодня выдуть в космосе огромный пузырь из стекла, покрыть его слоем серебра, нарезать из него шестиугольников и собрать сразу в космосе зеркало самого огромного телескопа?
381 454817
>>4816
Зеркало телескопа намного сложнее, чем кусок стекла с зеркальным напылением. До сих пор даже рублевый стаканчик из пластика на МКС сделать не могут.
382 454818
>>3914
Афигеть, особенно про такие кометы с афелием огромным, спс
383 454819
А если сделать надувной космический лифт. Допустим через каждый километр трос поддерживают дирижабле.
384 454820
>>3949
РЕЗИНКА
Е
З
И
Н
К
А

Длинная, она растянется и плавно утянет за собой искусственный спутник к астероиду
385 454822
>>3972
Ядерная боньба сильно проще и гораздо, гораздо меньше риск, что что-то пойдет не так и метеорит упадет на Китай
386 454823
>>3973
А кто туда захочет лететь после такого https://ru.m.wikipedia.org/wiki/Сквозь_горизонт ?
387 454827
>>4370

> Что за пределами твоих 8 гб ОЗУ на компе


address out of bounds
388 454828
>>4431
page file, программа даже не заметит, что больше, чем есть ОЗУ, жрет оперативы
389 454830
>>4539
Падажжите, а для чего лазеры в телескопах? Инопланетян подстреливать?
390 454834
>>4830
там метеорологическая хуйня для детекции искажений атмосферы за счет создания искусственной звезды. Это для супер дорогих телескопов с коррекцией изображения.
391 454848
>>4819
С падением давления катастрофически падает грузоподъемность аэростатов. Человек поднимался на 41 километр, шар сам по себе на 50 с чем-то. Выше уже практически невозможно.
392 454868
>>4848

>50 км.


Но ведь этого уже достаточно. Дальше лифт поддерживается центробежными силами.
393 454949
>>4820
С резинкой нужны будут в буквальном смысле тысячи километров троса, чтобы удержать максимальное ускорение в пределах нормы.
394 454950
>>4868
Центробежными силами он поддерживается с 36 тысяч километров, а вернее, на 36 тысячах километров должен быть его центр масс.
395 454951
Когда-то давно, в прошлой жизни, занимался наблюдением переменных и заподозренных в переменности блеска звезд. Сейчас эта тема актуальна вообще, кто-то угорает по живым наблюдениям, составлением графиков, все такое?
396 454952
>>4868
А ближе что, не поддерживается?
397 454953
>>4834
Ааа
398 454954
>>4949
Печалька, я тоже надеялся на фишку с астероидом, даже не предполагал таких подвохов
399 454955
>>4954

Обычное дело, если знаешь про банджи-джампинг, про торможение трением, возможно, даже в играх типа Just Cause нечто подобное проделывал, то сразу хочется применить что-то в этом роде для космических проблем.

Вся проблема в том, что у человека просто нет интуитивного понимания квадратичного роста чего бы то ни было, а требуемый тормозной путь и выделяемая в процессе кинетическая энергия растут именно квадратично. Кажется, что если со знакомых скоростей типа сотни км/ч можно затормозить резинкой в прыжке или просто тормозами на автомобиле за какую-нибудь сотню метров, то и для космических скоростей потребуется всего несколько километров троса.

А на деле, разумеется, хуй бы там, при росте скорости в сто раз все становится сложнее не в сто раз, а в десять тысяч.
400 454961
>>4951
Нет. Зачем? Если только поаутировать.
401 454974
>>3949
Охладить можно жидким газом, да можно придумать всякое. Тем более на один раз
402 454975
Почему гравитация влияет на время? То есть я понимаю, что оно вроде как связано с пространством но как?
И я могу понять искривление пространства, так как оно ощутимо вроде как, ну то есть оно везде, вроде как фундамента для всего. Я нахожусь в пространстве, планеты, галактики. Но вот время я не понимаю. Как, где оно находится, я даже своими словами не могу описать, что такое время. Оно просто есть и всё.
403 454979
>>4975

>оно вроде как связано с пространством


Временная координата - одна из координат пространства-времени, наряду с пространственными координатами, согласно специальной теории относительности.

>Почему гравитация влияет на время


Изучай общую теорию относительности. Там все ответы!
404 454981
>>4974

>Охладить можно жидким газом


А когда он нагреется можно его стравить через клапан и получить ускорение!
405 454982
>>4979

>Там все ответы!


Но никаких ответов там нет.
406 454983
Привет, аноны, два года назад примерно по различным новостным ресурсам бегала инфа о FRB 121102. Тогда теоретически объяснили это тем что куски плазмы могли выстроиться в ряд по направлению к земле и заслать радиоволны. Сам я не сильно шарю, почитываю время от времени этот тред. И в копании инфы не слишком силен, но может кто-то слышал/знает/владеет инсайдами/строил теории о данном явлении? Внеземной интеллект выходит на связь или ряд случайностей? А может фейк? Поделитесь и/или скажите, что думаете о поводу таких вещей
407 454984
>>4983
а, ну кстати и неск дней назад писали что-то, провафлил, натыкался только на 2017 статьи
408 454986
>>4983

> Внеземной интеллект выходит на связь или ряд случайностей? А может фейк?


Ни то, ни другое, ни третье. FRB 121102 - единственный FRB, который наблюдали мало того, что с нескольких радиотелескопов (точно Аресибо и Паркс), так еще и в другом диапазоне что-то нашли. Про куски плазмы в ряд хуета какая-то, которую я не понял, про внеземной интеллект - ещё большая хуета. Просто какое-то не катастрофическое, но мощное астрономическое событие типа вспышек магнитара. Там теорий, на самом деле, с десяток актуальных, я все не помню, астрономам нужно больше данных, чтобы определиться.
sage 409 454996
>>4982
Значит, ты их плохо искал.
410 454997
>>4996
Все правильно анон говорит. Никакого времени ирл, как физической величины, не существует, люди придумали его для удобства.
411 454998
>>4997
Это только ты так думаешь
412 455012
>>4974
Лень считать подробно, но если прикинуть на пальцах, то нагрев килограмма жидкого азота на 50 градусов поглотит всего ~100 кДж тепла, его испарение еще 200 кДж, и нагрев получившегося газа ну пусть даже на 1000 градусов даст еще около 1 МДж.

Короче, даже в идеальном варианте твоим жидким азотом можно поглотить всего 10% той энергии, которая уходит только на его собственное торможение, даже речи не идет про то, чтобы окупить хотя бы собственный вес.

Придумать всякое, говоришь, можно? Ну придумай, как за несколько десятков секунд поглотить и сбросить три килограмма тротилового эквивалента на килограмм массы корабля, и учти, что любая твоя охладительная система добавит еще массы, и еще тепла.
413 455117
>>4955

> выделяемая в процессе кинетическая энергия


Еще и атмосферы нет, чтобы ее отвести:(
414 455209
>>4975

>я даже своими словами не могу описать, что такое время


Ты уверен, что тебе нужен этот тред? Может лучше в философию? Кант подойдет с его размышлениями об априорных суждениях и антиномиях чистого разума, решение которых невозможно по его мнению нащупать эмпирически. Если тебе нужно определение на стыке технически-философского, то попробуй упороться по программируемой вселенной. Просто воспринимай пространство-время как функциональную систему, математическое поле с численным выражением координат для любой внутренней единицы. Человеки - переменные, не способные использовать время свободно. Привязаны к нему как к опорной шкале. Как ящерки или кто там, жившие в двумерном мире. Понятие вперед назад и вправо влево им знакомо. Вверх вниз уже нет.
image.jpeg1,3 Мб, 758x1200
415 455216
Посоны
Вышло
FIRST MAN
Говорят кинцо супер
416 455217
>>5216

>Вышло


Ага, 3 месяца назад.
417 455218
>>5217
Блу-рей сами знаете где появился только вчера-сегодня, до этого только качество для говноедов было
418 455220
>>5216

> First Man


> Человек на Луне



Что за ебаная болезнь у пидорашкинских переводчиков выковыривать из своей жопы хуй пойми что вместо того, чтобы переводить названия как есть? Ну что за мразь. Здесь даже никакой игры слов нет, просто бери и пиши "первый человек".
419 455221
>>5220
Двачую
420 455222
>>5216
Дак вот, я смотрю уже - там Нил говорит - на орбите Земли, при docking с другим кораблём, чтобы его догнать, надо уменьшить thrust, чтобы увеличить скорость, потому что уменьшив thrust ты попадёшь на более низкую орбиту - и это контринтуитивно.

Такие дела
421 455223
>>5220

>первый человек



Авторские права на словосочетание принадлежат РПЦ.
422 455232
>>5222
Real story кстати, на Gemini 4 именно так сближение с мишенью провалилось.
423 455252
>>5218
Я в кино сходил как бы. Но да, фильм мне зашел, хочу пересмотреть в оригинале, тем более на сцене стыковки с Адженой ходил поссать и проебал весь экшн.
sage 424 455270
>>5252

>2k18


>не сходить поссать до фильма

425 455271
>>5270

>2019


>ходить в кино


Вы там совсем тутуру?
426 455272
>>5271

>2019


>хикковать дома


>разговаривать помойными мемесами из бурятской 2d-мазни


Хмммм...
Sloupokpokemon.jpg9 Кб, 220x260
427 455284
>>1413 (OP)
Может ли звезда или прожорливый квазар быть настолько яркими, что излучают в невидимом для человеческого глаза спектре, а в видимом не излучают нихуя? Типа, подлетаешь к нему, а там пусто, включил спектрограф и охуел.
428 455287
>>5272
Как начнете ноомальное кинцо снимать, тогда и приходите.
429 455294
>>5284
Чисто теоретически больцмановское распределение это позволяет, на практике такое хорошо работает для холодных объектов в основном, а у горячих остается хвост и в оптике. Другое дело что часто наши инструменты достаточно чувствительны, чтобы увидеть пик в рентгене, но их не хватает для хвоста в оптике.
430 455336
>>5232
Да, глянул щас
https://en.wikipedia.org/wiki/Gemini_4
NASA engineers had not yet worked out the idiosyncrasies of orbital mechanics involved in rendezvous, which are counter-intuitive. Simply thrusting the spacecraft toward the target changed its orbital altitude and velocity relative to the target. When McDivitt tried this, he found himself moving away and downward, as the retrograde thrust lowered his orbit, increasing his speed.

Как я понял из слова retrograde, его корабль был впереди цели, если смотреть по направлению вращения вокруг Земли, поэтому thrust по направлению к цели его орбиту снижал
431 455357
>>5336
то есть они полетели тупо наобум, не озаботившись подумать, как на орбите маневрировать?
432 455410
>>5357
Поддвачиваю вопрос
433 455463
>>5410
НЕ наобум. Все было рассчитанно ещё за 5 лет до этого.
434 455487
Поясните за поверхность звезд-гигантов. Я знаю, что она неправильной формы, но насколько быстро колышутся эти вздутия?
436 455534
>>5496
Про большие конвективные ячейки то я знаю. А вот с какой скоростью они бурлят, известно?
437 455559
>>5534
В каких единицах скорость бурления и колышения вздутия тебе нужна? Тут их впервые разглядели на фото только, лол. у тебя оче специфический вопрос просто. Зависит от звезды, есть разные гиганты - желтые, белые, синие, супер, мега гига нигга...где границу между ячейками проводить?
вот бы зонд отправить и заснять это дерьмо у Сириуса
438 455567
>>5559
Как быстро по сравнению с Солнцем?
439 455622
>>5567
Чем больше ячейка, тем медленнее. Если размером с солнечную вспышку то и скорость такая же.
440 455670
>>5622
Все равно нихуя непонятно.
Ладно, попробую переформулировать вопрос.
Допустим, мы рядом с такой звездой. На космическом корабле или на планете, неважно. Мы будем видеть, как эти пузыри движутся, если будем смотреть на звезду, ну в течение минуты например? Или она будет статичной, как на снимке из твоего видоса?
49750s.jpg185 Кб, 807x558
441 455673
>>5670
Даже если они будут двигаться 10000 км в час, ты этого движения на фоне звезды не заметишь, лол. Размер звезды такой, что даже если ячейка будет двигаться "быстро", то по отношению к общему размеру звезды это будет незаметно. Как бы гигантские циклоны на земле тоже двигаются быстро, но с космической станции это движение не заметно без супер сильного зума на маленьких облаках.
Я не понимаю зачем тебе эта скорость так важна?
442 455675
Какие суперспособности я получу, если залезу в коллайдер в момент разгона частиц?
443 455676
>>5675
Ты получишь кочергой по ебалу от сторожа.
444 455677
>>5675
Станешь пидарасом
445 455678
>>5677
А если он уже стал, ну до коллайдера ещё?
446 455679
>>5678
Станешь суперпидарасом
447 455683
Что имеют ввиду, когда говорят - аппарат рассчитан на N лет работы? Если прекращение работы связано со случайной поломкой, а не расходом горючего, уставанием ритэга и т.д.

Рассчитывают срок безотказной работы с вероятностью 90 (или сколько?) процентов?
448 455687
>>5675
Проиграл с вопроса. Молодец
449 455689
>>5673

> Я не понимаю зачем тебе эта скорость так важна?


Ну непонятно же, как ты будешь эту звезду вблизи видеть. Как неподвижное бесформенное облако или как колышущееся бесформенное облако?
450 455690
>>5683
Надо различать расчетный (изготовителем) срок эксплуатации и гарантированный (гарантийный) срок эксплуатации. Первый из расчетов и испытаний рожают, он чисто информативный и никому ничего не должен, второй имеет под собой гарантийные обязательства перед заказчиком. Подозреваю, что гарантийные условия в контаркте обсуждаются, если пропишут случайные поломки и повреждения от комаров-гептилососов - будут компенсации, не пропишут - не будут.
мимо хуй
1534585795118826829.jpg153 Кб, 910x600
451 455694
>>5675
Интересный вопрос.
Из канала откачивают воздух.

Если эксперимент начался, и ты откроешь болгаркой какой-нибудь люк техобслуживания, воздух проникнет туда, пучок тормознется, эксперимент остановят и ты получишь пизды.

Если ты заныкаешься там заранее с дыхательным аппаратом, то пучок будет тормозиться о твою тушку и ты скорее всего сдохнешь от рака. Но перед тем как сдохнуть, получишь пизды. Но это не точно.
https://ru.wikipedia.org/wiki/Бугорский,_Анатолий_Петрович

Возможно также, что электромагнитное поле высокой плотности сделает из тебя еще большего дауна.
452 455695
>>5683
Смотрят, сколько твёрдо и чётко могут прожить критические системы аппаратуса работая как надо - сколько износа вынесет та или иная ебулда прежде чем у неё случится УСТАЛОСТЬ, на сколько хватит дури у солнечных батарей, примерно так чувствуют сколько вреда системам нанесёт космическая радиация с течением времени, всё в таком духе. Всё это проверяется расчётами и наземными испытаниями. Потом из всей хуйни считают на каком сроке вероятность отказа КРАЙНЕ МАЛА (точно не помню сколько у них там по требованиям, но меньше указанных тобой 10%, вроде 2% или что-то такое).
Так получается расчётный срок службы, своего рода ставка очка инженерами, что в этот срок аппарат будет работать без обосрамса, если конечно на сборке йобы чертежи не переворачивали вверх тормашками и не использовали вместо отвёртки сваезабойную машину.
453 455709
>>5694
Не от рака. Просто поджарится что нибудь внутри тела. Сам же пишешь, пучок будет тормозиться о тушку
454 455710
sage 455 455711
>>5710
Мало того что не туда ответил, так ещё и вкинул уже вкинутое. Гоните меня, насмехайтесь надо мной.
456 455714
>>5689

>как ты будешь эту звезду вблизи видеть.


Вблизи лучше не смотреть на неё, все равно деталей не различишь, а ослепнешь, если будешь без фильтра смотреть. Думаешь ты разглядишь пятна на нашем Солнце невооруженным глазом?
457 455715
>>5714
Я по крайней мере разгляжу, круглое оно или щербатое, а так же движутся щербины или нет. Можно через фильтр смотреть в конце концов. Ну и вообще красные гиганты тусклее светят же? Или нет? Забыл уже.
458 455716
>>5715
Смотря где ты смотреть будешь - с поверхности планеты, или из космоса, с какой планеты с какой атмосферой, как близко планета к звезде, как близко звезда к горизонту, если ты в космосе то тоже как близко ты к звезде - там доебаться много переменных, которые ты не уточняешь, еще тебе говорят что красные гиганты тоже разные бывают. Прост установи SpaceEngine и побалуйся там.
459 455717
>>5716
Ну с планеты допустим. На таком расстоянии, чтобы видно хорошо было без увеличения, но и не на полнеба. Меня интересует именно то, будет ли облако неподвижное, или эти конвективные ячейки все же будут как то двигаться, чтобы было видно невооруженным глазом?

> Прост установи SpaceEngine


Там неподвижные, я видел уже.
460 455727
>>5717
Да не будет ничего двигаться для невооруженного глаза, вот если ты камеру с фильтром поставишь и таймлапс сделаешь, то тогда заметишь колебания. Представь что волна двигается со скоростью 10000 км в час - если граница за час пройдет 100000 км по поверхности, это не составит даже 0.00001% от диска звезды. Ты понимаешь что гигант он огромный, он внутри себя орбиту меркурия и венеры может вместить, а это миллионы километров.
461 455728
>>5709

>Не от рака.


Тут все от плотности энергии пучка зависит. А не от ее величины.
В пучке БАКа все же настолько мала масса х количество частиц, что они просто прошьют тебя навылет. Дальше два крайних варианта, хотя возможны промежуточные:
Если больше некоего порога - острая лучевая болезнь, "смерть под лучом", гугли.
Если меньше - небольшая лучевка, а потом начнется рак, потому что атомы твоего организма наионизировались по самое не балуйся, а ДНК превратилось в говно. А если пучок нейтронный, тоже все интересно, можешь стать носителем изотопов, но ненадолго и с теми же последствиями.
462 455729
>>5727
То есть получается, что в небе будет светить не диск, а едакая клякса?
463 455731
>>5694

>Бугорский


хуяси
464 455732
>>5729
Думаю для твоего глаза будет диск, на который ты все равно не сможешь смотреть. Если ты поставишь фильтр, то будешь видеть возможно некоторые неровности, как протуберанцы на солнце у нас есть, но так ты будешь видеть эти гранулы. И может они будут жить долго, там будут свои циклы активности. Может раз в месяц или раз в год или десятилетие будут значительные изменения - всплытие новых гранул, затопление, слияние, раздвоение, помутнение? а иногда может отдельные гранулы будут становиться ярче и ты сможешь заметить неравномерность освещения от звезды, как будто одна грань более яркая чем другие? В любом случае изменения будут медленными, невооруженным глазом и до заката ты ничего не будешь видеть скорее всего да и не будет желания пялиться на звезду и получать ожоги сетчатки без цели.
465 455734
>>5732
А на закате неровности будет лучше видно?
466 455737
>>5734
зависит от влажности в атмосфере и вообще условий, но свет на закате путешествует через больше слоев атмосферы, так что ты сможешь без боли в глазах смотреть на звезду и увидеть что-то необычное. Может сможешь увидеть, что "солнце больное", не является идеальным кругом и не светится равномерном, как будто бы хмурится и рожа на нём, если у тебя парейдолия включится.
468 455769
>>4804
Все видим. Девятая планета не доказана.
469 455771
Допустим, я волшебник и могу управлять массами без проблем.
Я массу атмосферы венеры сбросил нахуй на Юпитер, оставив только примерно земную атмосферу на Венере, раскрутил ее. Свободного кислорода нет, взять неоткуда кроме земли, да?
Есть ли в солнечной системе достаточно комет чтобы на венеру накидать достаточно воды чтобы было столько же как и на земле?
470 455808
>>4979

>Временная координата - одна из координат пространства-времени, наряду с пространственными координатами, согласно специальной теории относительности.


https://habr.com/post/120066/
471 455836
>>5675
Суперспособность - быстрая смерть.
Не рекомендую, кстати, брат попробовал.
472 455838
>>5694

> Если ты заныкаешься там заранее с дыхательным аппаратом, то пучок будет тормозиться о твою тушку и ты скорее всего сдохнешь от рака.


Нет, у него просто кончится воздух раньше, чем туннель откачают до нужного вакуума.
473 455875
>>5808

>vixra.org/pdf/1106.0026v1.pdf


>vixra.org



Кек, статья с помойки для сайнс-фриков. Иди с этим в /сцы
15462958964170.png29 Кб, 569x571
474 455902
1) При каких условиях на планетах с атмосферой сможет создаться смерч (если вообще сможет) такой силы, чтобы человека, внезапно оказавшего под иглой основного тела смерча выкинуло бы нахуй за пределы атмосферы?
Вариант для Марса, как я понимаю, не рассматривается из-за разряжённости атмосферы, а вот планеты типа Земли или Венеры меня уже интересуют.
2) Узнал, что есть такие штуки как ФОНОНЫ. Это как фотоны, только не от мира света, от мира звука. Несколько раз внимательно вчитывался в статью на википедии, но так и не понял, шо эт за хуйня и как работает.
Можем ли мы построит фонопластины по аналогии с фотопластинами и заряжаться энергией от солнечного звука? Можно ли вообще слушать всё подряд через космос. раз фононы передают звук, хотя там нет волн, которые звук передают?
475 455905
А есть у нашего любимого Солнца какое-нибудь хитровыебаное название, типа этих HD YC 123-3498-1337γχ? Понятно что оно не нужно, но если бы оно было, можно же универсально по характеристикам звезды составить? Просто чтобы выебнуться
476 455923
Не совсем тупой вопрос, но хз где спросить. Кто может посоветовать аль подсказать какой-нибудь телескоп, чтобы не только луну рассматривать и планеты солнечной системы? И вообще рассказать, какие марки лучше, на какие характеристики смотреть и прочее.
477 455929
>>5923>>429065 (OP) — вот тут спроси. Тред временно в спячке, так как зимой через 100% облачность нихуя не понаблюдать и постить не о чем, но если задать вопрос, то ответят.
478 456008
Что будет, если северный магнитный и южный магнитный полюса совпадут?
Такое возможно?
479 456011
>>5902
Фононы - это дыры в кристаллической решетке
480 456012
>>5905

> типа этих HD YC 123-3498-1337γχ?


Эти названия присваиваются по телескопам и прочему астрономическому оборудованию.
Например, звёзды WISE - это звезды, которые нашли с помощью инфракрасного телескопа WISE
481 456013
>>6008
Невозможно. Можно только приблизить их друг к другу. Гугли подковообразные магниты.
482 456050
В аполлоне обнаружили говно и никто из присутствующих не признавался, а могли ли быть что в капсулу насрали на земле?
1969.05.18Apollo10LunarModuleRendezvous.jpg641 Кб, 2457x2457
483 456063
>>6050
Кто о чём, а двачер о говне.
Нет, потому что воняло бы при посадке экипажа в капсулу, что унюхал бы наземный персонал, который при этом процессе присутствует без скафандров.
Нет, потому что капсула просто мизерного объёма, там незаметно насрать маловероятно, особенно на Земле. По той же причине говно появилось бы в поле зрения почти сразу после вывода на орбиту - единственные места, где может находиться говно в условях земной гравитации, это в тыльной части капсулы (которая на Земле вообще "пол" капсулы) за спинками кресел космонавтов или в нижних ящиках, которые открываются задолго до следующей причины.
Нет, потому что говно начало гонять по капсуле на шестой день полёта, а шесть дней не замечать говно находясь в шестикубовом помещении может только совсем одичавший хикка, который срёт прямо на стул с хуями дрочёными, на котором сидит.
И всё это в случае покака с идеальными для орбитальной диверсии характеристиками, без следовой подливы или кашицеобразной консистенции.
484 456117
>>6063
им не ссыкотно было на этой хуйне из фольги и трубок куда-то лететь? и ведь если что-то сломается, то всё, мучительная смерть от удушья
485 456149
В соседнем треде красивые картинки, наведшие меня на вопросы о цвете небо.
Может ли быть ночное небо на атмосферной обитаемой планете синим в самые тёмные часы, а не чёрным? Чем это может быть обусловлено? Более плотной и толстой атмосферой?
И да, может ли быть плотная (и пригодная для дыхания) атмосфера в межпланетном космосе в пределах двойной планеты? Какого цвета будет это межпланетное небо?
От чего вообще зависит цвет неба других планет?
486 456153
>>6050

>аполлоне обнаружили говно


Спектакль для говнозависимых.
>>6117
Не ссыкотно, как и для экипажа Челленджера.
AS09-20-3104.jpg866 Кб, 2860x2890
487 456156
>>6117
Конечно, каждый спейсонавт наверняка в какой-то мере поссыкивает - буквально в нескольких сантиметрах от него за парой листов металла и изоляции находится мгновенный и бесповоротный пиздец, а его жизнь зависит от правильной работы огромного количества сложных йоб.
Однако первый пункт в требованиях при приёме на работу в космо/астронавты - стальные яйца, особенно в те времена, особенно в лунных миссиях. Настоящий космохуй должен полностью осознавать, в насколько рисковое дерьмо он лезет, и как говорил кстати кто-то из лунных вроде бы космохуёв, до определённой степени бояться нужно - потому что там, где совсем нет страха, обычно имеет место излишняя самоуверенность, а вот этого вот в космосе не надо.
488 456158
>>6149

> От чего вообще зависит цвет неба других планет?


От состава атмосферы, очевидно же!
На Марсе в атмосфере много частичек пыли с поверхности, поэтому небо там красноватое.
На Титане из-за метановой атмосферы все оранжевое.
489 456159
>>6063
А вот когда аполлон 11 прилунился, там ведь астронавт, который вышел первым, сначала выкинул мешок с мусором. А что было в том мешке? Говно?
1437422813-as11-40-5850hr.jpg885 Кб, 2349x2373
490 456160
>>6159
Да. Если быть точным, то миссии Аполлонов оставили на Луне 96 пакетов (внутри этих мешков) с уриной, говном и блевотиной. Кроме объектов твоих эротических фантазий там были ещё пустые упаковки из под космоеды, юзанные салфетки, полотенца и прочий гигиенический хабар.
491 456169
>>6160
А малафья была? Ну они же дрочили как-то.
492 456172
>>1413 (OP)
1. Как невесомость влияет на образующихся сперматозоидов?
2. Почему в невесомости выпрямляется позвоночник? И выпрямится ли он у старой бабули?
493 456173
>>6169
Там не уединиться, ты чего, только если на брудершафт. Хотя, тот кто оставался на орбите мог тихонько передернуть на Луну.
494 456188
Зачем наса пилит миллиарды на sls, зачем покмок маняфантазирует о сверхтяже? Ведь уже сейчас можно летать двухпуском, просто закидывать бак с топливом на орбиту и лететь куда угодно. Даже маск себе проблем добавил со старшипом, кроме земли плотная атмосфера есть только на титане, специализированная консерва для астероидов/спутников куда перспективнее. Можно даже многоразовый аппарат было запилить мкс-луна, а огурцов поднимать/спускать на тех же союзах.
495 456193
>>6172

> Почему в невесомости выпрямляется позвоночник? И выпрямится ли он у старой бабули?


Потому что Земля притягивает тебя с силой mg, но твердая поверхность не дает тебе упасть к ее центру, в итоге, когда ты стоишь, тебя придавливает к этой поверхности и сжимает твой позвоночник.
В невесомости ты постоянно падаешь, и нет твердой поверхности, к которой тебя бы прижимало.
496 456201
>>6188

>Зачем наса пилит миллиарды на sls


>Ведь уже сейчас можно летать двухпуском


Прикинь, SLS Block 1 двухпуском выведет столько же, сколько выводил Сатурн V. Если пускать одноразовыми Фальконами, то даже без учета систем сближения и стыковки, то даже шестипуск вытянет меньше, чем один Сатурн. При том, что Аполлон был приспособлен только для коротких полетов с непродолжительной высадкой.

>Даже маск себе проблем добавил со старшипом, кроме земли плотная атмосфера есть только на титане


У Марса в данном контексте достаточно плотная атмосфера. Скажем, Curiosity из 5,8 км/c об атмосферу погасил ~5,3. Если бы вместо атмосферы использовались двигатели, то на каждую тонну аппарата потребовалось около 5,25 тонны топлива. Если же гасить двигателями только последние 0,5 км/c, то на тонну аппарата потребуется всего около 175кг. Это без использования паращюта. В обоих случаях я привожу минимум, без какого-либо резерва на выбор места посадки и компенсацию ускорения свободного падения.
497 456203
>>6201
А, криво написал.

>Curiosity из 5,8 км/c об атмосферу погасил ~5,3.


Это он тепловым щитом гасил. На скорости около 0,5км/c открылся паращют.
498 456218
>>6156

>в нескольких сантиметрах от него бесповоротный пиздец


Чож тогда в подводники такой охуенный конкурс не проводят, а берут всех подряд?
499 456225
>>6218
Бля, да ты охуел?! Сравнил залупу из фольги, мотающую круги по орбите с заранее заданными параметрами движения и на месяцы вперед расписанным режимом корректировок, и дуру со стальными сантиметровыми стенками, которая не погружается дальше нескольких минут возврата на поверхность в режиме пузырика. Не, у самого дедов брат служил подводником по призыву, и я не умаляю заслуг и тягот, но это ебать совсем не то.
45874653256.jpg59 Кб, 326x326
500 456227
>>6218
Начнём с того, что в подлодке хотя бы можно изолировать отсеки, совершить экстренное всплытие, с определённой глубины можно выбраться через торпедный аппарат, а на некоторых судах есть даже спасательные капсулы иногда даже рабочие. А знаешь, что можно сделать для спасения при полной разгерметизации капсулы экипажа на орбите Луны? А вот нихуя.
Кроме того, весь экипаж АПЛ не должен знать орбитальную механику, уметь в расчёты и стыковку, действовать во всех возможных и невозможных нештатных ситуациях, знать все системы космического корабля на зубок, да ещё и научной хуйнёй заниматься - простому матросу хватит знаний по специальности и навыков обращения со шваброй и огнетушителем, а об остальном пусть думают мичманы, офицеры и командир.
Я утрирую конечно, потому что на АПЛ отбор всё-таки строже, чем в надводный флот, да и все они обязаны знать матчасть АПЛ и уметь в борьбу за живучесть, но если ты порешь хуйню, то почему мне нельзя.
image.png45 Кб, 523x324
501 456246
Мой тупой вопрос: мы ТЕОРЕТИЧЕСКИ смогли бы считать информацию об окружающем пространстве за горизонтом событий чёрной дыры? Приложу схему. Сам спейсшип движется с очень быстрой скоростью, так что провалится в горизонт только его датчик.
nikitosina.jpg41 Кб, 670x251
502 456247
>>6246
Нет, гравитационные силы разорвут космический зонд ещё на подлёте.
503 456249
>>6246
И квантовым методом вроде же не научились инфу отправлять
504 456250
>>6247
А если сверхмассивная?
>>6246
Теоретически твоя теория звучит ровно также как и "просто берешь и без задней мысли считываешь". Что ты считываешь, какую информацию? В чем заключается "квантовый метод"? С запутанными частицами сразу нахуй.
505 456251
>>6247
С чего вдруг? Приливные силы у горизонта событий могут быть минимальными - зависит от размера ЧД
506 456314
Если на сферическом астероиде подпрыгнуть со скоростью, скажем, 5 м/с, то с такой же скоростью и приземлишься. А что с астероидами неправильной формы? Возможно ли там подпрыгнуть и разбиться нахуй?
507 456317
>>6246
Датчик провалится и не вернется. И электроны в проводах из него тоже на спейсшип не вернутся, по которым ты собираешься инфу передавать.
508 456333
>>6251
Это не поможет тебе вернутся из сингулярности.
509 456347
>>6314
Не, если космическое говно недостаточно массивное, чтобы принять форму ехидного колобка, то там сильной гравитации не будет.
510 456362
>>6250
Почему нахуй, мы же не о практической реализуемости говорим, а о теории. Спутанность работает через горизонт событий? Или неизвестно (учитывая, что квантмех не стыкуется с СТО)?
511 456377
>>6362
Потому что через запутанность нельзя передать никакой информации.
512 456383
>>6362
В ОТО - нельзя, и в этом состоит один из ее парадоксов, показывающий ее несовместимость с квантовой механикой. Предполагается, что в квантовой теории гравитации информацию из ЧД можно извлечь, анализируя ее хокинговское излучение.
513 456398
>>6314
Теоретически да. Это по идее то же самое, что спрыгнуть с высокой горы.
514 456564
Почему космонавты, двигаясь по орбите, не испытывают перегрузок и их не прижамает к стенкам мкс при "поворотах"? Да, я знаю что сила не прикладывается, но направление движения-то меняется. Центрифуга меня в блин расплющит, а они там наверху катаются и им норм.
515 456567
>>6564
Потому что на них действуют те же самые силы, что и на саму МКС, даже если бы стенок станции не было, траектория их движения вокруг Земли ничуть бы не изменилась — насколько изменяется направление движения станции, настолько же отклоняется и направление движения космонавта.

В центрифуге центростремительное ускорение приложено только к самой центрифуге, а не к тебе, поэтому стенки капсулы постоянно относительно тебя ускоряются и плющат в лепешку. Нет относительного ускорения — нечему оказывать силу, нет и перегрузок.
516 456569
>>6564
думаю, уравновешивается гравитацией
мимогуманитарий
517 456573
>>6567
В центрифуге сама кабина придает мне ускорение, но моя туша хочет двигаться не по кругу, а по касательной к этому кругу, отсюда меня и вдавливает в кресло. И даже когда центрифуга прекратит набирать скорость и будет вращаться с постоянной без ускорения, меня все равно будет вжимать в кресло. На орбите же постоянно меняется и скорость движения возьмем для примера сильно эллиптическую орбиту и направление. Почему при прохождении перигилия, например, космонавтов точно так же не вжимает в стенки?
518 456580
>>6564
Я думаю, что испытывают. Это ведь по сути то же самое, как испытывать перегрузки в поворачивающем авто или автобусе.
519 456582
Поясните за искуственную гравитацию с помощью вращающегося тороидального колеса. Я слышал, что помимо прижимной силы там будет действовать сила Кориолиса. Каким образом она будет воздействовать на людей?
520 456588
>>6564

>Почему космонавты, двигаясь по орбите, не испытывают перегрузок и их не прижамает к стенкам мкс при "поворотах"


Прижимает ещё как. С какой перегрузкой прижимает догадаешься?
521 456611
>>6573

>Почему при прохождении перигилия, например, космонавтов точно так же не вжимает в стенки?


Блять, тебе законы Кеплера хоть о чем-нибудь говорят? Ты подразумевал перигей, я надеюсь. Смари кароч. Движение по орбитальной траектории, не столь важен эксцентриситет, имеет две составляющие. Инерциальную, ту самую, которая создает центробежное ускорение, и гравитационную. Первая космическая на том и завязана, что центробежное ускорение на высоте 400 км, например, равно силе земного притяжения на этой высоте, 8.8м/с2. Отсюда и берется невесомость на МКС, ну или микрогравитация, суть не в терминах. Так вот в гипотетической центрифуге радиусом в 6700 км, которую разгонят, а на конце у нее будет капсула с людьми внутри, разгонят до периода оборот в полтора часа. В ней люди ощутят "вдавливание" в стенки, с тем самым ускорением 8,8. Потому что система центрифуга и стержень, соединяющий ее с осью, и люди - это раздельные по приложению сил системы. В поле же тяготения планеты и капсула, и люди в ней - неразрывные части одной системы приложения сил. До сих пор непонятно?
522 456617
523 456621
>>6617
Почему Россия не может делать свою многоразовую ракету? Был же проект Байкал-Ангара.
524 456661
>>5902

> При каких условиях на планетах с атмосферой сможет создаться смерч (если вообще сможет) такой силы, чтобы человека, внезапно оказавшего под иглой основного тела смерча выкинуло бы нахуй за пределы атмосферы?


>Вариант для Марса, как я понимаю, не рассматривается из-за разряжённости атмосферы, а вот планеты типа Земли или Венеры меня уже интересуют.


Не может. При ебический силы ветрах когда человека может швырять отрывает асфальт от земли, но это внизу атмосферы.
Чтобы тебя вообще за атмосферу выкинуло - это надо чтобы сама атмосфера уже улетала еще быстрее, чтобы тебя за собой утащить. А это утащит и часть коры. Это катаклизм уровня массивного столкновения с крупным небесным телом вроде какой-нибудь цереры.
525 456662
>>5923

>хз где спросить


Специально есть тред >>429065 (OP)
526 456663
>>6621
Нерентабельно
527 456664
>>6050
Не признались потому, что никто и не знал, чей пакет раскупорился, они же все в пакетики срали и складывали бережно какашки, чтобы потом обмазываться ученым дать покушать на анализы. Да и за борт их в отличие от мочи не выкинешь особо.
528 456665
>>6117
Вообще-то, не мучительная. Удушье наступает от избытка СО2, а не недостатка кислорода. В случае быстрой разгерметизации - быстрая потеря сознания за десяток секунд.
Тебе не сыкотно на этой хуйне из алюминия и сверхбыстровращающихся лопастей летать? Ведь если что-то сломается, то всё, ты разбился.
Тебе не сыкотно на этой хуйне из железа и горючего куда-то ездить? Ведь если что-то сломается то всё, ты разбился.
529 456666
>>6149
Читай про Релэевское рассеяние. Оно отвечает за изменение цвета атмосферы, если она прозрачна. Если в ней взвесь говна какого-нибудь, то так >>6158
530 456670
>>6201

>паращюта


>>6203

>паращют


Тут совсем не "Щ", анон, а вполне себе "Ш".
531 456671
>>6582
При небольшом радиусе у тебя на голову будет воздействовать меньшая сила, чем на ноги.
Представь что ты поднимаешься по лестнице к центру тора - в центре у тебя ноль гравиатции. Так и тут - при небольшом радиусе разница в воздействии будет ощутима.
532 456699
Кто-нибудь перекат будет делать? Какого рода хотите перекат?
533 456700
>>6699
С пузырем альбукерке, конечно же.
534 456702
>>6700
С альбукерке уже было 10 тредов назад. Ладно, надо с чем-нибудь перекатить уже, а то так и утонем.
535 456703
>>6702
Тогда с жопой протона.
8768657.png29 Кб, 980x643
536 456705
Дано: есть качели-подобный механизм, у которого есть Плечо1, Плечо2 и центр, разделяющий плечи, приподнятый на Высоту. Анон стоит на П2. На конец П1 со скоростью V1 под углом У падает астероид/метеорит. Анона возьмём массой 100кг для ровно счёта и представим, что он не по-мясному упругий, а монолитно твёрдый, как кусок металла.
Вопрос: Какими должны быть параметры П1, П2, В, У и V1, чтобы такая установка V2 стала достаточно, чтобы смогла благополучно доставить анона в космос/на околземную орбиту/сделать искуственным и временно живым спутником планеты/отправить за пределы солнечной системы?
537 456715
Увеличится ли женская грудь в невесомости? Или же уменьшится?
538 456716
>>6715
Должна приподняться и визуально стать пышнее.
Стикер512x512
539 456717
>>6716
А может ли большая грудь мешать дышать при перегрузках 5G и выше? Неужели все женщины в космосе будут досками без доек

алсо, кто-то должен перекатиться тред
541 456722
>>6717
Тебе и без сисек будет трудновато дышать при таких перегрузках.
542 456723
>>6721
Раз уж сегодня затмение было, пусть будет отдаленно тематический перекат.
543 456763
>>6671
Я про нормальный радиус. 4 км, или сколько там должно быть по расчетам. Ну так что с силой кориолиса? Сильно она будет мешать?
544 456766
>>6721
Почему этот Аристарх такой недовольный?
545 457050
>>6665

>Тебе не сыкотно на этой хуйне из алюминия и сверхбыстровращающихся лопастей летать? Ведь если что-то сломается, то всё, ты разбился.


Нет, самолёт может сесть даже при отказе всех двигателей. Поломка чего-то в воздухе обычно не катастрофична.

>Тебе не сыкотно на этой хуйне из железа и горючего куда-то ездить? Ведь если что-то сломается то всё, ты разбился.


Нет, она просто остановится в 99% случаев. Если космический аппарат заглохнет, тебе пиздец.
546 458065
>>7050

>Если космический аппарат заглохнет, тебе пиздец.


Nyet.
Dm
Нам бы выпить перед стартом,
D Gm
Но другие помешают,
C F
Лишь бы старая "Семерка"
C F
Отработала свой план.
D
Если даст отказ "вторая",
Gm
Мы в Алтае иль в Китае,
Eb Dm
Если "третья" откажет -
A Dm
Здравствуй Тихий океан.
14190814866300.jpg56 Кб, 604x406
547 459157
>>6246
С ЧД вообще всё не так однозначно.

Современная физика учит, что всякое взаимодействие передаётся через бозоны-переносчики полей, и происходит это не быстрее скорости света. Жопа в том, что:
1. Масса ЧД скрыта под горизонтом событий.
2. Каким-то образом эта масса передаёт информацию о своём местоположении внешним гравитирующим объектам.
3. Хотя ни фотон, ни гипотетический гравитон не должны быть способны вылетать из-под горизонта, ведь скорость гравитационных волн равна скорости света.
Вменяемых разрешений этого парадокса я ещё не находил.
Тред утонул или удален.
Это копия, сохраненная 22 мая 2019 года.

Скачать тред: только с превью, с превью и прикрепленными файлами.
Второй вариант может долго скачиваться. Файлы будут только в живых или недавно утонувших тредах. Подробнее

Если вам полезен архив М.Двача, пожертвуйте на оплату сервера.
« /spc/В начало тредаВеб-версияНастройки
/a//b//mu//s//vg/Все доски